0% found this document useful (0 votes)
27 views53 pages

EKLT AILET Mock - 3 - Solutions

The document discusses concerns raised by geologists about potential biases and censorship in the GeoGPT chatbot, which is backed by Chinese funding. It highlights issues related to transparency, state censorship, and the implications of using AI models with inherent biases. The document also emphasizes the significance of recent discoveries in ancient malaria DNA, showcasing the transformative impact of modern techniques on understanding historical diseases.
Copyright
© © All Rights Reserved
We take content rights seriously. If you suspect this is your content, claim it here.
Available Formats
Download as PDF, TXT or read online on Scribd
0% found this document useful (0 votes)
27 views53 pages

EKLT AILET Mock - 3 - Solutions

The document discusses concerns raised by geologists about potential biases and censorship in the GeoGPT chatbot, which is backed by Chinese funding. It highlights issues related to transparency, state censorship, and the implications of using AI models with inherent biases. The document also emphasizes the significance of recent discoveries in ancient malaria DNA, showcasing the transformative impact of modern techniques on understanding historical diseases.
Copyright
© © All Rights Reserved
We take content rights seriously. If you suspect this is your content, claim it here.
Available Formats
Download as PDF, TXT or read online on Scribd
You are on page 1/ 53

LL.B.

ADMISSION TEST – 2025


LIFE TIME (LT) AILET MOCK Solution – 02

1. Correct Answer: (a) potential biases due to its Chinese backing.


Reference Line: "Geologists have raised concerns about potential Chinese censorship and bias in a
chatbot being developed with the backing of the International Union of Geological Sciences (IUGS)."
Explanation:
(a) potential biases due to its Chinese backing.
The primary concern raised by geologists revolves around potential censorship and bias arising from
GeoGPT's Chinese backing. This issue is critical because it could influence the integrity and objectivity
of the geological information provided by the chatbot. Given the international nature of geoscientific
research, any form of bias could skew results and lead to misinformation. The emphasis on Chinese
involvement suggests a worry that the information might be selectively presented or altered to align
with specific narratives or policies. Hence, option (a) is the correct answer.
(b) the possibility of incomplete geological data.
The passage does not mention concerns about the completeness of geological data. Incomplete data
would imply that the chatbot fails to provide a comprehensive overview of geological information, which
is a different issue from censorship and bias. Since the focus is on transparency and potential
manipulation of information, this option does not address the core issue highlighted by the geologists.
Hence, option (b) is not the correct answer.
(c) issues related to its user interface and accessibility.
User interface and accessibility issues would pertain to how easy or difficult it is for users to interact
with GeoGPT. However, the passage specifically discusses concerns related to potential censorship and
bias, not technical or usability problems. Therefore, while user interface and accessibility are important
aspects of any software, they are not the primary concerns in this context. Hence, option (c) is not the
correct answer.
(d) the high cost of its implementation and use.
The passage does not discuss the financial aspects of implementing or using GeoGPT. Concerns about
cost would involve discussions about funding, budget allocations, and financial sustainability, none of
which are mentioned. The focus remains on the integrity and reliability of the information provided by
the chatbot, making financial considerations irrelevant to the main concern. Hence, option (d) is not the
correct answer.

2. Correct Answer: (d) underlying biases in the AI models used.


Reference Line: "One of those who had tested a pre-release version of the chatbot claimed that GeoGPT
had 'serious issues around a lack of transparency, state censorship, and potential copyright
infringement.'"
Explanation:
(a) lack of adequate training data.
The passage does not indicate that the issues with GeoGPT stem from a lack of training data. Adequate
training data is crucial for developing an effective AI model, but the concerns raised are specifically
about transparency, censorship, and copyright infringement. These issues suggest problems with the
Page 1 of 53
content and the ethical implications of its use rather than the quantity or quality of the data used for
training. Hence, option (a) is not the correct answer.
(b) insufficient testing and quality assurance processes.
While testing and quality assurance are essential to developing a robust AI system, the passage does not
imply that the problems with GeoGPT arose from inadequate testing. Instead, the issues mentioned
relate to ethical and legal aspects, such as transparency and censorship, which would not necessarily be
resolved through more rigorous testing alone. Therefore, this option does not address the root cause of
the concerns. Hence, option (b) is not the correct answer.
(c) interference from international regulatory bodies.
There is no mention of interference from international regulatory bodies in the passage. Regulatory
interference would imply external constraints or mandates impacting the development or deployment
of GeoGPT, which is not discussed. The concerns are internally focused on the AI models' content and
the potential biases introduced by the developers. Hence, option (c) is not the correct answer.
(d) underlying biases in the AI models used.
The passage highlights concerns about transparency, state censorship, and potential copyright
infringement, which are indicative of underlying biases in the AI models. These biases could result from
the data used, the training process, or the developers' influence. Given that these issues were prominent
in the pre-release version, it suggests that the AI models' inherent biases were a significant factor in the
initial problems with GeoGPT. Hence, option (d) is the correct answer. ases in the AI models, which
include Alibaba’s Qwen. Hence, option (d) is the correct answer.

3. Correct Answer: (a) skeptical


Reference Line: "Geologists have raised concerns about potential Chinese censorship and bias."
Explanation:
(a) skeptical.
The writer presents the concerns raised by geologists and provides specific examples of potential
censorship and bias. This indicates a skeptical attitude towards the claims made by the developers of
GeoGPT. The skepticism is evident in the detailed discussion of the issues and the doubts about the
system's transparency and objectivity. Hence, option (a) is the correct answer.
(b) supportive.
A supportive attitude would involve endorsing or justifying the potential censorship and bias, which is
not the case here. The writer reports the concerns critically and does not express support for the
practices or the developers' claims. Instead, the focus is on highlighting the potential issues and the
skepticism surrounding the project. Hence, option (b) is not the correct answer.
(c) indifferent.
Indifference would imply a lack of concern or interest in the potential censorship and bias. However, the
writer clearly outlines the concerns and discusses them in detail, indicating a vested interest in the
issue. The tone and content of the passage show that the writer is neither indifferent nor dismissive of
the concerns. Hence, option (c) is not the correct answer.
(d) alarmed.
While the writer does express concern, the tone is more skeptical than alarmed. Being alarmed would
suggest a heightened sense of urgency and distress, which is not conveyed in the passage. The writer
presents the information in a measured and analytical manner, focusing on the potential implications
rather than reacting with alarm. Hence, option (d) is not the correct answer.

4. Correct Answer: (b) The chatbot exclusively uses data from Chinese sources.

Page 2 of 53
Reference Line: "GeoGPT is aimed at geoscientists and researchers, particularly in the global south, to
help them develop their understanding of earth sciences by drawing on swaths of data and research."
Explanation:
(a) GeoGPT aims to aid geoscientists in the global south.
The passage explicitly states that GeoGPT is aimed at geoscientists and researchers, particularly in the
global south, to help them develop their understanding of earth sciences. This goal is a key aspect of the
project, highlighting its intention to support scientific development in regions that may lack resources.
Hence, option (a) is not the correct answer.
(b) The chatbot exclusively uses data from Chinese sources.
The passage does not state that GeoGPT exclusively uses data from Chinese sources. While the project
is largely Chinese-funded and involves Chinese developers, it is implied that the data and research used
span a broader scope. The focus is on geoscience information, which likely includes international data.
Hence, option (b) is the correct answer.
(c) GeoGPT is still under development and not yet publicly released.
The passage confirms that GeoGPT is still under development and has not been released to the public.
This ongoing development status is crucial, as it indicates that the project is still being refined and that
any current issues may still be addressed before its release. Hence, option (c) is not the correct answer.
(d) The underlying AI for GeoGPT includes Meta’s Llama and Alibaba’s Qwen.
The passage mentions that GeoGPT uses both Meta’s Llama and Alibaba’s Qwen as part of its underlying
AI. This combination of AI models is significant in understanding the technological foundation of
GeoGPT and the potential sources of bias and other issues. Hence, option (d) is not the correct answer.

5. Correct Answer: (b) Both statements are true but 2 is not the correct explanation of 1.
Reference Line: "Problems with GeoGPT have been largely solved, but the team will be working to
improve the system even more."
Explanation:
(a) Both statements are true and 2 is the correct explanation of 1.
While both statements are true, the resolution of the issues does not explain the initial criticism. The
initial criticism was due to transparency and censorship concerns, which have been addressed but were
not caused by the resolution process. The resolution is a response, not an explanation. Hence, option (a)
is not the correct answer.
(b) Both statements are true but 2 is not the correct explanation of 1.
Both statements are accurate: GeoGPT faced criticism for transparency and censorship issues, and the
team is working on resolving these problems. However, the resolution process does not explain the
initial criticism; it is a response to it. This distinction makes option (b) the correct answer.
(c) Statement 1 is true but statement 2 is false.
Statement 2 is true according to the passage, which mentions that the problems have been largely
solved, with ongoing improvements. Therefore, this option incorrectly assesses the truthfulness of
statement 2. Hence, option (c) is not the correct answer.
(d) Statement 1 is false but statement 2 is true.
Statement 1 is true as per the passage, which discusses the criticism GeoGPT faced. Thus, this option
incorrectly assesses the truthfulness of statement 1. Hence, option (d) is not the correct answer.

6. Answer: (a) transformative


Reference line: "The team uncovered 36 individuals who had been infected with malaria across 26
archaeological sites on five continents."
Explanation:
Page 3 of 53
(a) Transformative - The discovery of ancient malaria DNA is transformative because it changes our
understanding of the history of this disease. By identifying malaria DNA in bones and teeth from ancient
remains, researchers have pushed back the known existence of the disease by thousands of years. This
discovery not only reveals the deep history of malaria but also shows the effectiveness of modern
techniques in uncovering historical diseases. The finding has profound implications for our knowledge
of ancient human health and disease transmission. Hence, option (a) is the correct answer.
(b) Trivial - Trivial means insignificant or of little importance. Given that the discovery has revealed new
information about the ancient prevalence of malaria, it is far from trivial. The identification of malaria
DNA in such ancient remains demonstrates the disease's long-standing impact on human populations,
providing valuable insights into historical disease patterns and human genetics. Therefore, option (b)
is not the correct answer.
(c) Tentative - Tentative implies that the findings are not definitive and are subject to further
verification. However, the study provided conclusive evidence by using advanced genetic techniques to
detect ancient malaria DNA, which strongly supports the researchers' conclusions. The robust nature of
these findings suggests that they are not tentative but rather well-substantiated. Thus, option (c) is not
the correct answer.
(d) Traditional - Traditional would indicate that the findings were expected or followed established
knowledge. The discovery of malaria DNA in ancient remains is novel and groundbreaking, pushing the
known history of the disease much further back than previously recorded. This innovative research does
not align with traditional findings but rather introduces new perspectives. Hence, option (d) is not the
correct answer.

7. Answer: (d) the physical traces were microscopic and scarce


Reference line: "Malaria is 'invisible archaeologically'. But with new techniques for analyzing genetic
samples, the researchers managed to find very small fragments of the parasitic DN (a)."
Explanation:
(a) The disease left no written records - While written records provide valuable historical context, the
main challenge in detecting ancient malaria lies in its physical traces. Written records alone cannot
reveal the presence of malaria in ancient remains, and the absence of such records does not directly
impact the ability to detect the disease archaeologically. Therefore, option (a) is not the correct answer.
(b) The ancient parasites could not be preserved - This option suggests that the malaria parasites
themselves did not survive in a form that could be detected. However, the discovery of DNA fragments
embedded in bones and teeth shows that remnants of the parasite can indeed be preserved over
millennia, albeit in microscopic quantities. Hence, option (b) is not the correct answer.
(c) The symptoms were non-existent - Malaria has distinct symptoms, but these symptoms do not leave
identifiable marks on bones or teeth that could be detected archaeologically. The issue is not the non-
existence of symptoms but the lack of visible, physical evidence that these symptoms might leave
behind. Thus, option (c) is not the correct answer.
(d) The physical traces were microscopic and scarce - The main difficulty lies in the microscopic nature
of malaria's physical traces. Malaria does not leave obvious signs in bones or teeth, making it "invisible
archaeologically." The researchers had to use advanced genetic techniques to detect the very small
fragments of parasitic DNA, highlighting the challenge of identifying such traces. Therefore, option (d)
is the correct answer.

8. Answer: (c) (3), (2), (4), (1)


Reference line: "But with new techniques for analyzing genetic samples, the researchers managed to
find very small fragments of the parasitic DN (a)."
Page 4 of 53
Explanation:
(a) (2), (3), (4), (1) - This sequence suggests that the development of techniques came before
recognizing the need to study ancient diseases. However, the logical progression starts with the
recognition of the need for study, which drives the development of new techniques. Hence, option (a) is
not the correct answer.
(b) (2), (4), (3), (1) - This sequence places sample collection before recognizing the need for study, which
is not a logical sequence. The need to study ancient diseases must be recognized before samples are
collected for analysis. Therefore, option (b) is not the correct answer.
(c) (3), (2), (4), (1) - This sequence correctly outlines the logical order: first, there is a recognition of the
need to study ancient diseases. This drives the development of advanced genetic analysis techniques.
Following this, samples are collected from various archaeological sites, leading to the discovery of
malaria DNA in ancient remains. Thus, option (c) is the correct answer.
(d) (3), (4), (2), (1) - This sequence incorrectly places sample collection before the development of
advanced techniques. The development of techniques is essential before sample collection can
effectively target the detection of ancient malaria. Therefore, option (d) is not the correct answer.

9. Answer: (c) the detection of ancient diseases using modern techniques


Reference line: "The new research pushes this date back dramatically, to 5,600 years ago in Germany—
the age of the oldest skeleton in which malaria was detected."
Explanation:
(a) The ability to travel across high altitudes in ancient times - While the ability to travel across high
altitudes is significant, it is not the main focus of the passage. The unprecedented aspect is related to
the detection of malaria using modern techniques, rather than the travel capabilities of ancient
populations. Hence, option (a) is not the correct answer.
(b) The trade routes connecting various regions in ancient history - Trade routes are mentioned in the
passage, but their existence is not unprecedented. The focus is on the new discovery of ancient malaria
DNA, which provides novel insights into the history of the disease, rather than the existence of trade
routes. Therefore, option (b) is not the correct answer.
(c) The detection of ancient diseases using modern techniques - This option correctly identifies the
unprecedented aspect of the discovery. The use of advanced genetic analysis to detect malaria DNA in
ancient remains is a groundbreaking achievement that extends our understanding of the disease's
history significantly further back in time than previously known. Hence, option (c) is the correct answer.
(d) The production of copper objects in lowland India - While the production of copper objects is
interesting, it is not the focus of the unprecedented discovery in the context of this passage. The main
discovery pertains to the detection of malaria, not the production of copper. Therefore, option (d) is not
the correct answer.

10. Answer: (a) they facilitated the spread of goods and diseases
Reference line: "The remains of the Nepali man were found near an ancient trade route crossing the
Himalayas."
Explanation:
(a) They facilitated the spread of goods and diseases - The passage highlights that trade routes not only
allowed for the exchange of goods but also facilitated the spread of diseases like malaria. The movement
of people along these routes led to the transmission of the disease across different regions, which is a
key point in the study. Hence, option (a) is the correct answer.
(b) They were established long before the discovery of malaria - The timing of the establishment of trade
routes is not the main focus. The significance lies in their role in spreading malaria, rather than their
Page 5 of 53
historical establishment relative to the discovery of the disease. Therefore, option (b) is not the correct
answer.
(c) They were limited to lowland areas prone to malaria - The passage mentions that trade routes
crossed various terrains, including high altitudes. They were not limited to lowland areas. The key point
is their role in spreading malaria, not their geographic limitations. Hence, option (c) is not the correct
answer.
(d) They were exclusively used for the exchange of metals - Trade routes were used for the exchange of
various goods, not just metals. The passage indicates the spread of both goods and diseases,
emphasizing the broader impact of these routes. Therefore, option (d) is not the correct answer.

11. Answer: (c) the necessity of innovation in historical disease detection


Reference line: "But with new techniques for analyzing genetic samples, the researchers managed to
find very small fragments of the parasitic DN (a)."
Explanation:
(a) Their privileged access to state-of-the-art technology - While access to advanced technology was
crucial, the focus is on the innovation and necessity of new techniques rather than the privilege of
access. The significance lies in the methodological advancements rather than access itself. Hence, option
(a) is not the correct answer.
(b) The failure of earlier archaeological methods - Earlier methods were limited in their ability to detect
microscopic traces of disease, but they were not failures. The new techniques represent an advancement
rather than a correction of past failures. Therefore, option (b) is not the correct answer.
(c) The necessity of innovation in historical disease detection - This option correctly highlights the
importance of developing new methods to detect diseases in ancient remains. The researchers' success
underscores the critical role of innovation in expanding our understanding of historical diseases. Hence,
option (c) is the correct answer.
(d) The importance of textual records in verifying ancient diseases - While textual records provide
context, the discovery relied on genetic analysis rather than textual verification. The innovation in
detection techniques is the primary highlight, not the role of textual records. Therefore, option (d) is not
the correct answer.

12. Answer: (b) the historical extent of human mobility and disease spread
Reference line: "The remains of the Nepali man were found near an ancient trade route crossing the
Himalayas. The archaeological site also contains copper objects produced in the lowlands of India,
where mosquitoes and malaria are common."
Explanation:
(a) The resilience of ancient human populations to extreme environments - While the ability to live in
extreme environments is notable, the significance in this context is more about disease spread. The
finding does not primarily highlight resilience but rather the historical movement and infection routes.
Hence, option (a) is not the correct answer.
(b) The historical extent of human mobility and disease spread - This option correctly identifies the key
significance. The discovery of malaria in the remains of a man who traveled across ancient trade routes
illustrates the widespread movement of people and the diseases they carried. This finding underscores
the interconnectedness of ancient populations. Therefore, option (b) is the correct answer.
(c) The early adaptation of humans to cold, high-altitude climates - The passage does not emphasize
adaptation to climates. Instead, it focuses on the spread of malaria and the movement of people along
trade routes. Hence, option (c) is not the correct answer.

Page 6 of 53
(d) The influence of trade on the development of early civilizations - While trade is mentioned, the
primary focus of the passage is on the spread of malaria. The significant finding is the historical
movement of the disease, rather than the broader influence of trade on civilizations. Therefore, option
(d) is not the correct answer.

13. Correct Answer: (a) Local political analysis


Reference Line: "With the INDIA bloc making significant gains against the BJP in Uttar Pradesh in Lok
Sabha polls, the by-elections in 10 assembly seats that have fallen vacant are likely to witness a keen
contest."
Explanation:
(a) Local political analysis
The passage provides a detailed account of the political dynamics in Uttar Pradesh, focusing on the
INDIA bloc's gains against the BJP and the upcoming by-elections in the state. It mentions the specific
parties involved, the number of seats, and the political implications of the elections. Such a detailed
discussion about local political events, parties, and election strategies indicates that the passage is most
likely a part of a local political analysis, providing insights and updates on regional political
developments. Hence, option (a) is the correct answer.
(b) International economic report
The passage does not address any international economic issues or provide any economic analysis. It is
entirely focused on the political scenario in Uttar Pradesh, mentioning specific political parties, local
elections, and their implications. There are no discussions on global markets, economic policies, or
international trade, making this option irrelevant. Therefore, option (b) is not the correct answer.
(c) Scientific journal
Scientific journals typically discuss research findings, experiments, and advancements in various
scientific fields. This passage, however, is centered on political events and election strategies in Uttar
Pradesh, with no mention of scientific studies or research. The language and focus are entirely political
rather than scientific, making this option unsuitable. Hence, option (c) is not the correct answer.
(d) Educational policy paper
Educational policy papers discuss issues related to the education sector, including policies, reforms, and
educational practices. This passage does not mention any educational topics or policies. It solely
discusses the political scenario and by-elections in Uttar Pradesh, making it clear that it is not related
to educational policies. Hence, option (d) is not the correct answer.

14. Correct Answer: (b) To inform about the political scenario and upcoming by-elections in Uttar
Pradesh.
Reference Line: "The date for the by-elections has not been announced yet by the Election Commission.
However, the Samajwadi Party (SP) and Congress have announced that they will contest together under
the INDIA bloc."
Explanation:
(a) To advocate for a new political party in Uttar Pradesh.
The passage provides information about the current political scenario, including the INDIA bloc's gains
and the BJP's preparations for the by-elections. It does not advocate for the creation of a new political
party. Instead, it focuses on existing parties and their strategies. The tone is informative rather than
persuasive towards forming a new party. Hence, option (a) is not the correct answer.
(b) To inform about the political scenario and upcoming by-elections in Uttar Pradesh.
The passage details the political developments in Uttar Pradesh, discussing the gains of the INDIA bloc,
the BJP's preparations, and the upcoming by-elections. It mentions specific parties, the number of seats
Page 7 of 53
affected, and the potential implications of the by-elections. The primary intent is to provide readers with
an update on the political situation and the upcoming electoral contest. Hence, option (b) is the correct
answer.
(c) To persuade voters to support the BJP in the upcoming by-elections.
The passage provides a balanced view of the political preparations of both the INDIA bloc and the BJP.
It does not show a bias towards any party or attempt to persuade readers to support the BJP. The focus
is on providing information rather than persuasion, indicating that the primary purpose is not to
influence voter behavior. Hence, option (c) is not the correct answer.
(d) To critique the performance of the INDIA bloc in recent elections.
While the passage mentions the INDIA bloc's gains, it does not critique their performance. Instead, it
objectively reports on their political activities and preparations for the by-elections. There is no negative
assessment or critique of the INDIA bloc's actions, which indicates that the passage's primary purpose
is not to criticize them. Hence, option (d) is not the correct answer.

15. Correct Answer: (d) The proactive approach of the BJP in facing the by-elections.
Reference Line: "The ruling Bharatiya Janata Party (BJP)-led National Democratic Alliance (ND(a) has
also begun its preparations, according to their leaders."
Explanation:
(a) The lack of organization within the BJP.
The passage indicates that the BJP, as part of the NDA, has already begun its preparations for the by-
elections. This suggests that the party is organized and proactive in its approach. The term
“preparations” highlights the BJP’s readiness and effort rather than a lack of organization. Hence, option
(a) is not the correct answer.
(b) The uncertainty surrounding the by-election dates.
While the passage mentions that the dates for the by-elections have not been announced, the focus on
“preparations” by the BJP and the NDA indicates that they are actively getting ready for the elections
despite this uncertainty. The term “preparations” underscores the efforts being made rather than the
uncertainty itself. Hence, option (b) is not the correct answer.
(c) The unimportance of the by-elections for the BJP.
The passage notes that while the numerical impact of the by-elections might be inconsequential for the
BJP due to its comfortable majority, it does not suggest that the by-elections are unimportant. Instead,
the BJP’s active preparations indicate that they view these elections as significant for maintaining their
political standing and morale. Hence, option (c) is not the correct answer.
(d) The proactive approach of the BJP in facing the by-elections.
The mention of “preparations” highlights that the BJP is taking a proactive approach to the upcoming
by-elections. The party is gearing up and making efforts to contest the elections with full strength,
demonstrating their commitment and strategic planning. This proactive stance is emphasized through
the mention of their preparations. Hence, option (d) is the correct answer.

16. Correct Answer: (c) "The ruling Bharatiya Janata Party (BJP)-led National Democratic Alliance
(ND(a) has also begun its preparations, according to their leaders."
Reference Line: "The ruling Bharatiya Janata Party (BJP)-led National Democratic Alliance (ND(a) has
also begun its preparations, according to their leaders."
Explanation:
(a) "The date for the by-elections has not been announced yet by the Election Commission."

Page 8 of 53
This sentence is a simple declarative statement that does not contain any participial phrases. It
straightforwardly states the fact about the by-election dates without using any modifying phrases.
Hence, option (a) is not the correct answer.
(b) "UP Assembly Special Secretary Brajbhushan Dubey told PTI that information about the vacancy on
10 seats has been sent to the Election Commission."
This sentence reports a statement made by an official and does not include any participial phrases. It
contains a clause that explains the action taken, but there are no participial modifiers present. Hence,
option (b) is not the correct answer.
(c) "The ruling Bharatiya Janata Party (BJP)-led National Democratic Alliance (ND(a) has also begun its
preparations, according to their leaders."
This sentence contains a participial phrase, “(BJP)-led,” which modifies “National Democratic Alliance
(ND(a).” The participial phrase provides additional information about the NDA, indicating that it is led
by the BJP. This is a clear example of a participial phrase used in the sentence. Hence, option (c) is the
correct answer.
(d) "As per the procedure, by-elections can be held on these seats within six months."
This sentence explains a procedural detail and does not include any participial phrases. It is a
straightforward statement about the timeline for the by-elections, without any modifying phrases.
Hence, option (d) is not the correct answer.

17. Correct Answer: (b) It will influence the morale and perception of both parties.
Reference Line: "While the by-election results will be numerically inconsequential for the BJP in the
state assembly, where it has a comfortable majority, they will impact the morale of both sides as any
further losses to the BJP will help the opposition SP and Congress to further cement their gains."
Explanation:
(a) It will change the overall number of seats significantly.
The passage explicitly states that the numerical impact of the by-elections will be inconsequential for
the BJP in the state assembly, where they already have a comfortable majority. Therefore, the impact is
not expected to significantly change the overall number of seats held by the BJP. Hence, option (a) is not
the correct answer.
(b) It will influence the morale and perception of both parties.
The passage mentions that while the by-election results will be numerically inconsequential, they will
impact the morale of both sides. Any further losses for the BJP will boost the opposition SP and Congress,
helping them to cement their gains. This indicates that the impact on morale and perception is a crucial
factor, making it more than just a numerical issue. Hence, option (b) is the correct answer.
(c) It will determine the leadership of the BJP.
There is no mention in the passage that the by-election results will determine the leadership of the BJP.
The focus is on the overall morale and perception rather than leadership changes. The BJP’s internal
leadership dynamics are not discussed in relation to the by-elections. Hence, option (c) is not the correct
answer.
(d) It will finalize the electoral strategy for the next general elections.
While the passage discusses preparations for the by-elections, it does not suggest that the results will
finalize the electoral strategy for the next general elections. The emphasis is on the immediate impact
on morale and the current political scenario rather than long-term strategic planning for future
elections. Hence, option (d) is not the correct answer.

18. Correct Answer: (a) The BJP is preparing to make significant efforts to win the by-elections.

Page 9 of 53
Reference Line: "BJP's state media in-charge Manish Dixit told that the 'NDA will contest the assembly
by-elections with full strength.'"
Explanation:
(a) The BJP is preparing to make significant efforts to win the by-elections.
The passage details the BJP's active preparations and commitment to contesting the by-elections with
full strength. Statements from BJP leaders indicate that the party is gearing up to face the elections
seriously, making significant efforts to regain its hold. This proactive approach highlights their
determination. Hence, option (a) is the correct answer.
(b) The BJP is indifferent to the outcome of the by-elections.
The passage does not support the idea that the BJP is indifferent to the by-elections. On the contrary, it
highlights the party’s preparations and efforts, indicating that they are taking the elections seriously
and are keen on performing well. Indifference is not suggested anywhere in the passage. Hence, option
(b) is not the correct answer.
(c) The BJP is planning to boycott the by-elections.
There is no indication in the passage that the BJP is planning to boycott the by-elections. The focus is on
their preparations and active participation, with leaders discussing their strategies and readiness.
Boycotting is not mentioned or implied. Hence, option (c) is not the correct answer.
(d) The BJP is confident of an easy victory in the by-elections.
While the passage mentions the BJP's preparations, it does not explicitly state that the party is confident
of an easy victory. The emphasis is on their readiness and efforts, rather than on a perceived guarantee
of success. The tone does not suggest overconfidence but rather a strategic approach to the elections.
Hence, option (d) is not the correct answer.

19. Correct Answer: (b) Illustrate the competitive nature of the upcoming by-elections.
Reference Line: "BJP's state media in-charge Manish Dixit told that the 'NDA will contest the assembly
by-elections with full strength.'"
Explanation:
(a) Highlight the BJP’s confidence in winning the by-elections.
The passage mentions the BJP's preparations but does not explicitly highlight their confidence in
winning. The focus is on their proactive approach and readiness rather than on their confidence. While
preparation suggests seriousness, it does not necessarily equate to highlighting confidence. Hence,
option (a) is not the correct answer.
(b) Illustrate the competitive nature of the upcoming by-elections.
The detailed discussion of both the INDIA bloc’s and the BJP's preparations for the by-elections
illustrates the competitive nature of the contest. The passage indicates that both sides are gearing up
for a keen contest, highlighting the efforts and strategies being employed. This illustrates the
competitive dynamics of the upcoming elections. Hence, option (b) is the correct answer.
(c) Criticize the BJP’s electoral strategies.
The passage does not contain any critique of the BJP’s electoral strategies. It objectively reports on the
preparations being made by the BJP and does not offer any negative assessments or criticisms of their
approach. The tone is informative rather than critical. Hence, option (c) is not the correct answer.
(d) Suggest that the BJP is at a disadvantage in the by-elections.
While the passage mentions that the by-elections will be a keen contest, it does not suggest that the BJP
is at a disadvantage. Instead, it discusses the preparations being made by the BJP to face the elections
with full strength. The emphasis is on their proactive approach rather than on any perceived
disadvantage. Hence, option (d) is not the correct answer.

Page 10 of 53
20. Correct Answer: (a) ‘inconsequential’
Reference Line: "While the by-election results will be numerically inconsequential for the BJP in the
state assembly, where it has a comfortable majority, they will impact the morale of both sides."
Explanation:
(a) ‘inconsequential’
The word "inconsequential" means having no significant impact or importance, which aligns with the
idea of being numerically insignificant. The passage mentions that the by-election results will be
numerically inconsequential for the BJP, indicating that the numerical impact is minimal. Substituting
‘inconsequential’ with ‘numerically insignificant’ maintains the same meaning. Hence, option (a) is the
correct answer.
(b) ‘impactful’
The word "impactful" means having a strong effect or influence, which is the opposite of numerically
insignificant. Replacing ‘impactful’ with ‘numerically insignificant’ would change the meaning of the
sentence, making this option incorrect. Hence, option (b) is not the correct answer.
(c) ‘significant’
The word "significant" means important or noteworthy, which is the opposite of numerically
insignificant. Substituting ‘significant’ with ‘numerically insignificant’ would completely change the
meaning, making this option incorrect. Hence, option (c) is not the correct answer.
(d) ‘competitive’
The word "competitive" describes a scenario where there is competition, which does not relate to
numerical significance or insignificance. Substituting ‘competitive’ with ‘numerically insignificant’
would not make sense in the context, making this option incorrect. Hence, option (d) is not the correct
answer.

21. Correct Answer: (a) Maintain their series of victories.


Reference Line: "While the SP and the Congress have expressed confidence that they will continue their
winning streak..."
Explanation:
(a) Maintain their series of victories.
The phrase "continue their winning streak" means to maintain an ongoing series of victories. In the
context of the passage, it refers to the confidence expressed by the SP and Congress in maintaining their
success in elections, suggesting that they aim to keep winning consecutively. This interpretation aligns
perfectly with the phrase’s meaning. Hence, option (a) is the correct answer.
(b) Initiate a new sequence of wins.
The phrase "continue their winning streak" does not imply starting a new sequence but rather
maintaining an existing one. The passage refers to the SP and Congress maintaining their existing series
of electoral successes, not beginning a new streak. Hence, option (b) is not the correct answer.
(c) Terminate their run of losses.
The phrase "continue their winning streak" suggests ongoing success and does not relate to ending a
series of losses. It implies that the parties have already been winning and aim to keep it that way, not
that they are trying to stop losing. Hence, option (c) is not the correct answer.
(d) Balance their victories and defeats.
The phrase "continue their winning streak" does not imply a balance between victories and defeats but
rather an unbroken series of wins. The SP and Congress are focused on maintaining their continuous
success in elections, not balancing their outcomes. Hence, option (d) is not the correct answer.

22. Correct Answer: (d) The BJP has strong local leaders in the vacant seats.
Page 11 of 53
Reference Line: "The ruling party is under pressure to regain its hold."
Explanation:
(a) The BJP has increased its majority in other states recently.
While an increased majority in other states might boost the BJP's overall confidence, it does not directly
address the specific pressure they face in Uttar Pradesh due to the gains of the INDIA bloc. The localized
pressure in Uttar Pradesh remains a concern irrespective of broader successes. Hence, option (a) is not
the correct answer.
(b) The opposition parties are facing internal conflicts.
Internal conflicts within opposition parties might weaken their collective strength, but it does not
necessarily reduce the pressure on the BJP to perform well in the by-elections. The BJP would still be
under pressure to maintain its hold and not lose further ground. Hence, option (b) is not the correct
answer.
(c) Voter turnout for by-elections is traditionally low.
Low voter turnout might affect the overall impact of the by-elections, but it does not eliminate the
pressure on the BJP to perform well. The significance of maintaining political morale and perception
among the electorate still holds, even with lower turnout. Hence, option (c) is not the correct answer.
(d) The BJP has strong local leaders in the vacant seats.
Having strong local leaders in the vacant seats would provide the BJP with a strategic advantage,
reducing the pressure to regain its hold. Strong candidates can enhance the party’s chances of winning,
thereby weakening the perception that the BJP is under significant pressure. Hence, option (d) is the
correct answer.

23. Correct Answer: (a) Analytical


Reference Line: "The stalled monsoon has led to an all-India deficit in monsoon rainfall of 20 per cent
between June 1 and June 18, heralding a sluggish start to the season responsible for 70 per cent of the
country’s total rainfall."
Explanation:
(a) Analytical
The passage presents detailed data and observations about the monsoon, its stalling, and the resultant
impact. This includes statistical information such as the 20% deficit in rainfall and the number of days
the monsoon has been stalled. The approach of dissecting the monsoon's behavior and providing
insights indicates an analytical tone, where the author is more focused on presenting facts and analysis
rather than expressing an emotional viewpoint. Hence, option (a) is the correct answer.
(b) Pessimistic
A pessimistic tone would convey a sense of hopelessness or negativity about the situation. While the
passage discusses the adverse impacts of the stalled monsoon, it does so in a factual manner without
indicating despair or hopelessness. The objective presentation of data suggests that the author is not
being pessimistic. Hence, option (b) is not the correct answer.
(c) Optimistic
An optimistic tone would highlight a positive or hopeful perspective. The passage does not provide any
hopeful predictions or positive spin on the situation; rather, it remains focused on presenting the
current state of affairs and the challenges posed by the stalled monsoon. Hence, option (c) is not the
correct answer.
(d) Apathetic
An apathetic tone would imply indifference or a lack of concern. The passage, however, is deeply
engaged with the details of the monsoon's progression and its impacts, indicating a vested interest in

Page 12 of 53
the topic. The detailed analysis shows concern and involvement, ruling out apathy. Hence, option (d) is
not the correct answer.

24. Correct Answer: (a) Cause: Unusual duration of monsoon break periods, Effect: Flash floods and
landslides in certain regions
Reference Line: "Some states like Sikkim, Arunachal Pradesh and Assam have also witnessed flash floods
and landslides due to sudden heavy rainfall events in the last few days."
Explanation:
(a) Cause: Unusual duration of monsoon break periods, Effect: Flash floods and landslides in certain
regions
The passage specifically mentions that unusual break periods in the monsoon have led to flash floods
and landslides in regions like Sikkim, Arunachal Pradesh, and Assam due to sudden heavy rainfall
events. This clearly establishes a cause (unusual monsoon break periods) and effect (flash floods and
landslides). Hence, option (a) is the correct answer.
(b) Cause: Early onset of the monsoon in Kerala, Effect: Normal monsoon progression
The early onset of the monsoon in Kerala is mentioned, but the passage clarifies that the monsoon has
been stalled, indicating abnormal rather than normal progression. This does not establish a direct
cause-and-effect relationship between early onset and normal progression. Hence, option (b) is not the
correct answer.
(c) Cause: Deficit in monsoon rainfall, Effect: Consistent temperatures in southern India
The passage does not link the rainfall deficit to consistent temperatures in southern India. In fact, it
mentions that southern India has had ample rainfall, suggesting variability rather than consistency.
Hence, option (c) is not the correct answer.
(d) Cause: Movement of monsoon trough, Effect: Excess rainfall in the northeastern states
The passage notes that the monsoon trough has not reached its normal position and therefore cannot
be responsible for the excess rainfall in the northeastern states. It is the stalling, rather than normal
movement, that is causing heavy rains. Hence, option (d) is not the correct answer.

25. Correct Answer: (a) To indicate a temporary cessation in monsoon progress


Reference Line: "The Bay of Bengal branch of the southwest monsoon has been stalled since May 31."
Explanation:
(a) To indicate a temporary cessation in monsoon progress
The term "stalled" is used to describe a situation where the progress of the monsoon has temporarily
stopped, as evidenced by the context of the passage discussing a halt in the monsoon's advance. This
suggests that the stoppage is not permanent but a pause in movement. Hence, option (a) is the correct
answer.
(b) To emphasize the long-term nature of the rainfall deficit
While the stalled monsoon contributes to a rainfall deficit, the term "stalled" itself does not emphasize
long-term implications. It specifically refers to a temporary halt, not a prolonged deficit. Hence, option
(b) is not the correct answer.
(c) To convey a sense of finality regarding the monsoon
"Stalled" suggests a pause or delay, not an end. The sense of finality is not conveyed by this word, as the
monsoon is expected to resume eventually. Hence, option (c) is not the correct answer.
(d) To suggest a minor delay in monsoon activity
The term "stalled" implies more than a minor delay; it indicates a significant pause in progress. The
passage details the considerable impact of this stalling, which is more severe than a minor delay. Hence,
option (d) is not the correct answer.
Page 13 of 53
26. Correct Answer: (a) Historical data shows similar stalling periods in previous decades.
Reference Line: "This is a stalling period of 19 days as of June 19, one of the longest in recent time,
according to data from the India Meteorological Department (IM(d)."
Explanation:
(a) Historical data shows similar stalling periods in previous decades.
If historical data reveals that similar stalling periods have occurred frequently in the past, it would
undermine the argument that the current prolonged stall is unusual. This would suggest that such stalls
are part of a recurring pattern rather than an anomaly. Hence, option (a) is the correct answer.
(b) The Arabian Sea branch of the monsoon frequently experiences short stalls.
The focus of the passage is on prolonged stalling. Mentioning that short stalls are frequent does not
address the issue of unusual prolonged stalls and thus does not weaken the conclusion. Hence, option
(b) is not the correct answer.
(c) Recent climate models predict prolonged stalling due to climate change.
This would support rather than weaken the conclusion, as it suggests that prolonged stalling is
becoming more common due to climate change, aligning with the passage's implications. Hence, option
(c) is not the correct answer.
(d) The current stall is predicted to end in the next week.
While this might offer immediate relief, it does not address the unusual nature of the prolonged stall up
to the present. It does not weaken the conclusion about the current period's unusual length. Hence,
option (d) is not the correct answer.

27. Correct Answer: (b) "The monsoon's progression is highly unpredictable and varies year to year."
Reference Line: "‘Break periods’ are a normal characteristic for the monsoon. But some of the recent
breaks have been much longer than normal."
Explanation:
(a) "The monsoon's early onset guarantees normal seasonal rainfall distribution."
The passage highlights that despite the early onset, the monsoon has stalled, leading to irregularities
and deficits in rainfall. This contradicts the idea that an early onset guarantees normal distribution.
Hence, option (a) is not the correct answer.
(b) "The monsoon's progression is highly unpredictable and varies year to year."
The author discusses the variability and unpredictability of the monsoon, including unusual break
periods and their impacts, indicating agreement with this statement. Hence, option (b) is the correct
answer.
(c) "Northern India will consistently receive more rainfall than southern India."
The passage mentions variability in rainfall patterns across regions, with southern India receiving
ample rainfall during the stall, which contradicts a consistent pattern favoring northern India. Hence,
option (c) is not the correct answer.
(d) "Flash floods are a sign of overall monsoon strength."
Flash floods are presented as a result of sudden heavy rains during unusual breaks, not as an indicator
of the overall strength of the monsoon. The passage does not support this interpretation. Hence, option
(d) is not the correct answer.

28. Correct Answer: (b) detrimental


Reference Line: "The stalled monsoon has led to an all-India deficit in monsoon rainfall of 20 per cent
between June 1 and June 18."
Explanation:
Page 14 of 53
(a) beneficial
The passage discusses the negative impacts of the stalled monsoon, including a 20% rainfall deficit and
adverse weather conditions, suggesting that these breaks are not beneficial to agriculture. Hence, option
(a) is not the correct answer.
(b) detrimental
The stalled monsoon has caused a significant rainfall deficit and affected temperatures, leading to
challenges in agricultural planning and crop growth. This indicates that unusual breaks are harmful.
Hence, option (b) is the correct answer.
(c) irrelevant
The detailed discussion of the impacts of the monsoon stalls on rainfall distribution and temperatures
indicates that these breaks are highly relevant to agricultural planning. Hence, option (c) is not the
correct answer.
(d) secondary
The passage presents the monsoon breaks as a primary concern, not a secondary one, due to their direct
impact on rainfall and agriculture. Hence, option (d) is not the correct answer.

29. Correct Answer: (a) The author would argue for increased investment in predictive technologies.
Reference Line: "This is a stalling period of 19 days as of June 19, one of the longest in recent time,
according to data from the India Meteorological Department (IM(d)."
Explanation:
(a) The author would argue for increased investment in predictive technologies.
The passage highlights the need for understanding and predicting unusual monsoon patterns, implying
that better predictive technologies could help manage the impacts of such variability. Hence, option (a)
is the correct answer.
(b) The author would likely dismiss the need for accurate predictions.
The emphasis on the unusual nature of the current stall and its impacts suggests that accurate
predictions are crucial, not dismissible. Hence, option (b) is not the correct answer.
(c) The author would insist that predictions are already sufficiently accurate.
The passage indicates that current prediction capabilities may not be sufficient, given the unusual and
prolonged nature of the monsoon stalls. Hence, option (c) is not the correct answer.
(d) The author would emphasize the importance of traditional forecasting methods.
There is no specific mention of traditional methods in the passage, and the discussion implies a need
for improved, possibly modern, predictive technologies. Hence, option (d) is not the correct answer.

30. Correct Answer: (c) "The weather patterns had shifted before the onset of the monsoon season."
Reference Line: "The Bay of Bengal branch of the southwest monsoon has been stalled since May 31."
Explanation:
(a) "The Arabian Sea branch of the monsoon frequently stalls for brief periods."
This sentence is in the present tense, not the past perfect tense. Hence, option (a) is not the correct
answer.
(b) "The monsoon has consistently shown unpredictable patterns."
This sentence uses the present perfect tense ("has shown"), not the past perfect tense. Hence, option (b)
is not the correct answer.
(c) "The weather patterns had shifted before the onset of the monsoon season."
This sentence correctly uses the past perfect tense ("had shifted") to indicate an action completed before
another past action ("the onset of the monsoon season"). Hence, option (c) is the correct answer.
(d) "The rainfall deficit continues to affect agricultural activities."
Page 15 of 53
This sentence is in the present tense, not the past perfect tense. Hence, option (d) is not the correct
answer.

31. Correct Answer: (d) Accelerated


Reference Line: "The Arabian Sea branch of the monsoon has been stalled since June 10, which is a
stalling period of nine days."
Explanation:
(a) Delayed
"Delayed" suggests a postponement in progress, similar to "stalled," where the monsoon is temporarily
halted. Hence, option (a) is a correct substitution.
(b) Halted
"Halted" means to bring to a stop, which aligns with "stalled" in indicating that the monsoon has
temporarily stopped moving. Hence, option (b) is a correct substitution.
(c) Interrupted
"Interrupted" implies a break or pause in continuity, which is consistent with "stalled" in indicating a
temporary cessation of activity. Hence, option (c) is a correct substitution.
(d) Accelerated
"Accelerated" means to increase in speed, which is the opposite of "stalled" and indicates movement
rather than a stoppage. Hence, option (d) is not a correct substitution.

32. Correct Answer: (a) To highlight the unusual weather patterns and their impact on various regions.
Reference Line: "The monsoon trough is an elongated low-pressure region that causes the rainfall
during the monsoon season over the country."
Explanation:
(a) To highlight the unusual weather patterns and their impact on various regions.
The passage details the unusual stalling of the monsoon, the resultant rainfall deficit, and the specific
regional impacts, indicating that the author's aim is to highlight these unusual patterns and their
consequences. Hence, option (a) is the correct answer.
(b) To argue that monsoon breaks are beneficial for certain areas.
The passage does not suggest that the monsoon breaks are beneficial; rather, it focuses on the negative
impacts such as rainfall deficit and adverse weather conditions. Hence, option (b) is not the correct
answer.
(c) To suggest that the monsoon is becoming more predictable over time.
The passage discusses the unpredictable nature of the monsoon, particularly the unusual and prolonged
breaks, indicating that predictability is not increasing. Hence, option (c) is not the correct answer.
(d) To promote the idea that southern India will always receive ample rainfall.
While the passage mentions that southern India has received ample rainfall during the stall, this is not
the main focus or aim. The aim is to discuss the unusual stalling and its broader impacts. Hence, option
(d) is not the correct answer.

33. Correct answer: (a) (1) and (2)


Reference lines: "In an age of mob scrolling, the sheer physicality of handling foldable paper has
currency... a gesture-habit that, above all, is empowering."
Explanation:
(a) (1) and (2): This option emphasizes that the act of reading a newspaper allows one to disconnect
from digital distractions, as well as being an act of empowerment and tradition. The passage supports
this interpretation with phrases like "In an age of mob scrolling, the sheer physicality of handling
Page 16 of 53
foldable paper has currency," suggesting that reading a physical newspaper offers a break from the
constant influx of digital information and provides a sense of empowerment and a connection to
tradition. The reference to "a gesture-habit that, above all, is empowering" further underscores the
empowering nature of this practice. Hence, options (1) and (2) are correct.
(b) (1) and (4): This option juxtaposes the idea of disconnecting from digital distractions with the notion
of the practice being outdated. The passage does not support the idea that reading a newspaper is
outdated; instead, it celebrates the physical and ritualistic experience of handling a newspaper. The
mention of "currency" in handling foldable paper implies that this practice still holds significant value
in contemporary times. Therefore, options (1) and (4) are not the correct answers.
(c) (2) and (3): This option suggests that reading a newspaper is both empowering and a method to stay
updated. While staying updated is a part of the experience, the passage places more emphasis on the
physical act itself as empowering and traditional, rather than just a means to gather information. The
phrase "a gesture-habit that, above all, is empowering" highlights the ritualistic and empowering nature
of reading a newspaper. Therefore, options (2) and (3) are not the correct answers.
(d) (3) and (4): This option contrasts staying updated with the practice being outdated. The passage
does not imply that reading a newspaper is outdated; it highlights the value of the physical experience
and the tradition associated with it. By focusing on the ritualistic aspect and the empowering nature of
reading a newspaper, the passage refutes the idea that it is merely an outdated practice. Therefore,
options (3) and (4) are not the correct answers.

34. Correct answer: (b) the ritualistic and tactile experience it provides
Reference lines: "The ritual begins with the crisp rustle of the paper as it unfolds... it's technology at its
most subtle."
Explanation:
(a) The comprehensive and diverse range of news: While a newspaper does offer a variety of content,
the passage emphasizes the experience of reading a physical newspaper rather than just the range of
news. The reference to the "canvas of images and words" and the "ritual begins with the crisp rustle of
the paper" underscores the tactile and ritualistic aspects that set the physical newspaper apart from
digital formats. Therefore, option (a) is not the correct answer.
(b) The ritualistic and tactile experience it provides: This option accurately captures the essence of what
the passage emphasizes. The act of unfolding the paper, the sound it makes, and the physicality of
handling the paper are highlighted as key benefits that distinguish the physical newspaper. The phrases
"a gesture-habit that, above all, is empowering" and "the crisp rustle of the paper as it unfolds" reinforce
this idea, making the tactile experience a unique and valued benefit. Therefore, option (b) is the correct
answer.
(c) The quick and instant access to updates: Instant access is more characteristic of digital formats, not
physical newspapers. The passage does not mention speed or immediacy as benefits of reading a
physical newspaper. Instead, it focuses on the leisurely and tactile nature of the experience, highlighting
how it contrasts with the fast-paced digital world. Therefore, option (c) is not the correct answer.
(d) The environmentally friendly nature of paper: The passage does not discuss the environmental
impact of newspapers. It focuses on the physical and ritualistic experience rather than any ecological
considerations, making the environmental aspect irrelevant to the unique benefit highlighted.
Therefore, option (d) is not the correct answer.

35. Correct answer: (a) that enhances human interaction and shared experiences
Reference lines: "It's technology at its most subtle: a shared experience... a link to everyone else."
Explanation:
Page 17 of 53
(a) That enhances human interaction and shared experiences: This option accurately reflects the
context in which "technology" is used in the passage. The phrase "It's technology at its most subtle: a
shared experience" indicates that the physical newspaper fosters interaction and shared experiences,
linking people through common topics of conversation without the need for digital means. The subtle
use of the term technology emphasizes the traditional and non-digital aspects of this experience.
Therefore, option (a) is the correct answer.
(b) That is advanced and digital: The passage uses "technology" in a non-digital sense, highlighting the
subtlety and traditional aspects of the physical newspaper. It contrasts with the typical association of
technology with digital advancements, suggesting a more nuanced and human-centric use of the term.
Therefore, option (b) is not the correct answer.
(c) That is obsolete and inefficient: The passage does not portray the newspaper as obsolete or
inefficient. Instead, it emphasizes its enduring value and the empowering, tactile experience it offers,
suggesting that it is still relevant and valuable. Therefore, option (c) is not the correct answer.
(d) That represents the future of communication: The passage focuses on the traditional and ritualistic
aspects of reading a newspaper, suggesting it as a valuable practice in the present rather than a futuristic
mode of communication. It highlights the continuity and enduring nature of this medium. Therefore,
option (d) is not the correct answer.

36. Correct answer: (b) Both statements are true but 2 is not the correct explanation of 1.
Reference lines: "a shared experience, dope for conversation, a link to everyone else without the bother
of actually meeting everyone else... invites a leisurely exploration"
Explanation:
(a) Both statements are true and 2 is the correct explanation of 1: While both statements are true,
statement 2 does not explain statement 1. The passage suggests that the sense of connection comes
from the shared experience and the common topics of conversation a newspaper provides, rather than
the leisurely nature of reading. Therefore, option (a) is not the correct answer.
(b) Both statements are true but 2 is not the correct explanation of 1: This option is accurate because
both statements are supported by the passage independently. The sense of connection is fostered by the
shared experience and common conversation topics, while the leisurely exploration refers to the way
one engages with the content of the newspaper. Therefore, option (b) is the correct answer.
(c) Statement 1 is true but statement 2 is false: Both statements are supported by the passage. The sense
of connection is highlighted through phrases like "a shared experience," and the leisurely nature of
reading is suggested by "invites a leisurely exploration." Therefore, option (c) is not the correct answer.
(d) Statement 1 is false but statement 2 is true: The passage supports both statements. Statement 1 is
true as the newspaper provides a shared experience and common topics for conversation. Statement 2
is true as it invites a leisurely exploration of the content. Therefore, option (d) is not the correct answer.

37. Correct answer: (a) the variety of content in a newspaper


Reference lines: "Each page turned is a journey through stories - of real people, local happenings, global
events..."
Explanation:
(a) The variety of content in a newspaper: This option correctly captures the meaning of "a journey
through stories" in the passage. The phrase "Each page turned is a journey through stories - of real
people, local happenings, global events..." highlights the diverse content that a newspaper offers. This
variety allows readers to explore different topics and stories, making the reading experience rich and
multifaceted. Therefore, option (a) is the correct answer.

Page 18 of 53
(b) The physical act of turning pages: While the physical act is mentioned, it is not the primary focus of
the phrase "a journey through stories." The passage emphasizes the content and variety of stories rather
than just the act of turning pages. The journey through stories implies a deeper engagement with the
content, not just the mechanics of reading. Therefore, option (b) is not the correct answer.
(c) The global reach of news: The passage mentions global events as part of the content, but "a journey
through stories" encompasses more than just the global reach; it includes local happenings and other
diverse content. The phrase implies a comprehensive exploration of various stories, both local and
global. Therefore, option (c) is not the correct answer.
(d) The convenience of reading: The passage does not emphasize convenience as a primary benefit.
Instead, it highlights the ritualistic and tactile experience of reading a newspaper and the variety of
content it offers. The focus is on the richness of the stories rather than the ease of access or convenience.
Therefore, option (d) is not the correct answer.

38. Correct answer: (b) The city was a canvas of diverse cultures and traditions.
Reference lines: "The broadsheet layout, a canvas of images and words..."
Explanation:
(a) The artist used a large canvas to create her masterpiece: This option refers to a literal canvas used
for painting, which is not the metaphorical sense used in the passage. The passage uses "canvas"
metaphorically to describe the layout of a newspaper filled with diverse elements. Therefore, option (a)
is not the correct answer.
(b) The city was a canvas of diverse cultures and traditions: This option uses "canvas" in a metaphorical
sense similar to the passage, representing a mixture of diverse elements. The passage refers to the
newspaper layout as "a canvas of images and words," highlighting the diverse content and how it forms
a cohesive whole. Therefore, option (b) is the correct answer.
(c) The football field, a canvas of green, was ready for the big match: This option uses "canvas" to
describe a literal field, which is not the metaphorical sense intended in the passage. The passage's use
of "canvas" refers to a variety of content, not a physical space. Therefore, option (c) is not the correct
answer.
(d) The novel's plot was a canvas for the author's vivid imagination: While this option uses "canvas"
metaphorically, it refers to imagination rather than a mixture of diverse elements, which does not align
with the passage's use. The passage emphasizes the diversity of content in a newspaper, not the creative
process of writing a novel. Therefore, option (d) is not the correct answer.

39. Correct answer: (b) a curated selection of news and entertainment.


Reference lines: "a canvas of images and words, headlines trickling into copies, invites a leisurely
exploration the regular gobbler of info can't afford... the daily dose of comic strips, crosswords and the
tongue-in-cheek in a curated space."
Explanation:
(a) A quicker method to gather news updates: The passage does not emphasize speed or immediacy as
a benefit of reading a physical newspaper. Instead, it highlights the ritualistic and tactile nature of the
experience, suggesting that the process is more leisurely and intentional compared to the rapid
consumption of digital news. Therefore, option (a) is not the correct answer.
(b) A curated selection of news and entertainment: This option accurately captures the essence of what
the passage describes. The reference to "a canvas of images and words, headlines trickling into copies,
invites a leisurely exploration the regular gobbler of info can't afford" and "the daily dose of comic strips,
crosswords and the tongue-in-cheek in a curated space" underscores the curated nature of the

Page 19 of 53
newspaper content, offering a balanced mix of news and entertainment. Therefore, option (b) is the
correct answer.
(c) An eco-friendly alternative to digital media: The passage does not discuss the environmental impact
of newspapers. It focuses on the physical and ritualistic experience rather than any ecological
considerations, making the environmental aspect irrelevant to the unique benefit highlighted.
Therefore, option (c) is not the correct answer.
(d) A personalized reading experience: While the experience is unique, the passage emphasizes the
communal and shared aspects of reading a newspaper rather than personalization. The idea of a "shared
experience" and a "link to everyone else" suggests that the benefit is more about a collective engagement
with curated content. Therefore, option (d) is not the correct answer.

40. Correct Answer: (d) Reduced job satisfaction and increased burnout
Reference Lines: "Greece has many challenges like low wages, high unemployment and a declining
population... working more stands in stark contrast to most of its European neighbors."
Explanation:
(a) Increased job satisfaction and morale: More workdays typically mean less time for personal life and
rest, which can lead to decreased satisfaction and morale. Employees might feel overworked and
stressed, reducing their overall happiness and productivity. The lack of time to unwind and spend with
family can contribute to feelings of dissatisfaction. Hence, option (a) is not the correct answer.
(b) No significant change in employee well-being: With the introduction of an extra workday, it's
improbable that well-being will remain unaffected. Increased work hours generally lead to higher levels
of stress and fatigue, impacting mental and physical health. The assumption that more work will not
affect well-being overlooks the importance of rest and personal time. Hence, option (b) is not the correct
answer.
(c) Enhanced work-life balance: More workdays typically disrupt work-life balance, making this option
less plausible. Work-life balance is about having enough time for both professional and personal life,
and an extra workday would reduce the time available for personal activities. This disruption can lead
to higher stress levels and reduced quality of life. Hence, option (c) is not the correct answer.
(d) Reduced job satisfaction and increased burnout: Increased workdays can lead to higher stress and
fatigue, aligning with common consequences of extended work hours. Employees might experience
burnout due to continuous work without sufficient rest, affecting their mental health and job
performance. This option aligns with the understanding that more work without adequate rest can lead
to negative outcomes. Hence, option (d) is the correct answer.

41. Correct Answer: (c) By increasing the work hours, contrary to the reduction trends
Reference Lines: "Companies in several countries like Germany, Belgium, France, the UK, Spain and
Iceland have been experimenting with different workweek models... squeezing the 40-hour week into
four days... or simply getting 100% of the work done in just 80% of the time."
Explanation:
(a) By aligning with the four-day workweek trends in Europe: Greece is moving in the opposite direction
by increasing workdays, not reducing them. Other European countries are experimenting with shorter
workweeks to improve productivity and work-life balance, which contrasts with Greece's approach.
Hence, option (a) is not the correct answer.
(b) By introducing a more flexible and shorter workweek: This option contradicts the increase in
workdays. Flexibility and shorter workweeks are designed to enhance employee well-being and
productivity, while Greece's policy aims at increasing the number of workdays, thus reducing flexibility
and potentially increasing stress. Hence, option (b) is not the correct answer.
Page 20 of 53
(c) By increasing the work hours, contrary to the reduction trends: Correct, as Greece is adding
workdays instead of reducing them. This increase in work hours stands in stark contrast to the
reduction trends in other European countries, which are focusing on improving employee satisfaction
and efficiency through fewer work hours. Hence, option (c) is the correct answer.
(d) By adopting similar workweek reductions as seen in Germany and Belgium: This option is incorrect
since Greece is not reducing work hours. Countries like Germany and Belgium are moving towards
shorter workweeks to promote a better work-life balance, whereas Greece is increasing the workweek,
indicating a different approach. Hence, option (d) is not the correct answer.

42. Correct Answer: (b) A continued struggle with economic reforms and workforce dissatisfaction
Reference Lines: "During their debt crisis, which started in 2009 and nearly got the country kicked out
of the eurozone, some lenders demanded that the Greeks work more. The country accepted bailouts
worth billions of euros that came with strict austerity measures."
Explanation:
(a) A potential resurgence of economic stability through increased labor output: This is speculative and
doesn't align with the historical dissatisfaction. While increased labor might aim to boost economic
output, the negative impact on employee well-being and satisfaction could undermine this goal.
Historical dissatisfaction with increased work demands suggests this approach may not lead to
economic stability. Hence, option (a) is not the correct answer.
(b) A continued struggle with economic reforms and workforce dissatisfaction: Correct, as history
shows the dissatisfaction with increased work demands. The new policy reflects ongoing challenges in
balancing economic reforms with worker satisfaction, indicating that Greece continues to grapple with
these issues. The historical context of resistance to longer workweeks supports this interpretation.
Hence, option (b) is the correct answer.
(c) An alignment with progressive labor practices seen in neighboring countries: The passage indicates
Greece is not aligning with these trends. Other European countries are adopting more progressive labor
practices, such as shorter workweeks, which contrast sharply with Greece's approach of increasing
workdays. This option does not accurately reflect the situation. Hence, option (c) is not the correct
answer.
(d) A likely shift towards more equitable labor conditions: Increasing workdays typically does not
equate to more equitable conditions. Equitable labor conditions often involve considerations for work-
life balance and employee well-being, which are not supported by extending the workweek. Therefore,
this option does not align with the described changes. Hence, option (d) is not the correct answer.

43. Correct Answer: (c) Greece adopts a stricter approach by increasing workdays
Reference Lines: "Greece though is taking another path. And this isn’t the first time the country has been
confronted with a six-day workweek."
Explanation:
(a) Greece's approach is more lenient and supportive of workers' rights: Increasing workdays is not
lenient. A lenient and supportive approach would likely involve measures to reduce work hours or
improve working conditions, which contrasts with the increase in workdays in Greece. Hence, option
(a) is not the correct answer.
(b) Greece aligns closely with other countries by reducing work hours: Greece is not reducing work
hours. Other countries are focusing on shorter workweeks to enhance productivity and employee well-
being, whereas Greece is moving towards longer workweeks, indicating a different approach. Hence,
option (b) is not the correct answer.

Page 21 of 53
(c) Greece adopts a stricter approach by increasing workdays: Correct, as Greece is increasing workdays
contrary to other European trends. This approach reflects a stricter stance on labor regulations, aiming
to increase labor output but potentially at the cost of employee well-being. Hence, option (c) is the
correct answer.
(d) Greece mirrors the flexible working models of its European neighbors: This option is incorrect since
Greece's model is not flexible. Flexibility in labor models typically involves reduced work hours or more
adaptable schedules, which is not the case with Greece's new policy of increasing workdays. Hence,
option (d) is not the correct answer.

44. Correct Answer: (a) It ensures these sectors remain competitive and attractive to workers
Reference Lines: "Tourism and food service industries are excluded from this new arrangement."
Explanation:
(a) It ensures these sectors remain competitive and attractive to workers: Correct, as maintaining a five-
day workweek can make these sectors more appealing. By not imposing the six-day workweek on
tourism and food service industries, Greece may ensure these sectors remain attractive to employees,
who might prefer more balanced work schedules. Hence, option (a) is the correct answer.
(b) It highlights a disregard for sector-specific needs and conditions: Exclusion indicates consideration
of specific sector needs. The decision to maintain the five-day workweek in certain sectors shows that
the government recognizes the unique demands and conditions of these industries. Hence, option (b) is
not the correct answer.
(c) It shows a uniform application of labor regulations across all sectors: The passage indicates a
differentiated approach. Uniform application would mean the six-day workweek applies to all sectors,
but the exclusion of tourism and food service suggests a tailored approach to different industry needs.
Hence, option (c) is not the correct answer.
(d) It underscores the government's priority on industrial and manufacturing sectors: While true, this
does not capture the significance for tourism and food service sectors specifically. The primary focus
might be on increasing productivity in industrial and manufacturing sectors, but this does not explain
the importance of maintaining a five-day workweek in the other sectors. Hence, option (d) is not the
correct answer.

45. Correct Answer: (d) A large and luxurious commercial development.


Reference Lines: "hoping to preserve the Canadian heritage site Ontario Place were dealt a serious blow
last week, when an Ontario court rejected a closely watched legal challenge to the provincial
government’s plans to build a mega-spa on one of the Modernist complex’s artificial islands."
Explanation:
(a) A modest recreational facility for the public.
The term 'mega-spa' implies a grand and extensive development. A 'modest recreational facility' does
not align with the term 'mega,' which denotes something large and impressive. A modest facility would
be small and simple, whereas a mega-spa suggests a significant, luxurious establishment. Therefore, this
interpretation is incorrect. Hence, option (a) is not the correct answer.
(b) A small-scale project focusing on environmental preservation.
A small-scale project focused on environmental preservation does not fit the context of a 'mega-spa.'
The prefix 'mega' suggests a large-scale development, which contrasts with a small-scale initiative.
Additionally, the context of the passage indicates that the development is controversial due to its impact
on the environment, further disproving this option. Hence, option (b) is not the correct answer.
(c) A traditional spa offering limited services.

Page 22 of 53
A 'mega-spa' indicates something beyond the scope of a traditional spa offering limited services. The
term 'mega' conveys the idea of something expansive and luxurious, with a wide range of services. The
contrast between 'mega' and 'limited services' makes this interpretation inaccurate. Hence, option (c)
is not the correct answer.
(d) A large and luxurious commercial development.
The term 'mega-spa' accurately describes a large and luxurious commercial development. The use of
'mega' implies scale and opulence, suggesting an extensive facility offering a wide array of amenities
and services. This aligns with the context of the passage, which highlights the development's significant
impact. Hence, option (d) is the correct answer.

46. Correct Answer: (c) The government proactively sought to conduct a thorough assessment of the
heritage site.
Reference Lines: "The suit had been filed in response to an attempt to scuttle a required assessment of
the heritage site prior to redevelopment by the government of Ontario premier Rob Ford."
Explanation:
(a) The government attempted to bypass a necessary evaluation of the heritage site.
This inference is correct as it aligns with the meaning of 'scuttle,' which suggests an attempt to avoid or
circumvent the required assessment. The passage clearly indicates the government’s effort to bypass
the environmental review. Hence, option (a) is the correct answer.
(b) The suit was a reaction to the government's effort to avoid an obligatory review.
This option correctly infers the meaning, as it mirrors the intent of the passage. The legal suit was indeed
a response to the government’s attempt to dodge the mandatory environmental assessment, which is
highlighted in the text. Hence, option (b) is the correct answer.
(c) The government proactively sought to conduct a thorough assessment of the heritage site.
This inference is incorrect because it contradicts the passage's content. The passage indicates that the
government tried to avoid the assessment, not conduct it proactively. The term 'scuttle' signifies an
effort to evade the assessment, not to carry it out thoroughly. Hence, option (c) is not the correct answer.
(d) The legal challenge was a response to the government’s plan to neglect a required evaluation.
This option correctly infers the passage's meaning, as the suit was indeed a reaction to the government's
plan to neglect the required environmental assessment. The term 'scuttle' indicates an effort to abandon
or circumvent the assessment. Hence, option (d) is the correct answer.

47. Correct Answer: (b) To emphasize its significance to the local community.
Reference Lines: "Critics of the redevelopment plan say it will not only destroy the landscape architect
Michael Hough’s original design but also wildlife habitats and local vegetation (including chopping
down more than 800 trees), all while privatising a well-loved public space."
Explanation:
(a) To highlight the commercial value of Ontario Place.
The term 'well-loved public space' emphasizes emotional and communal value rather than commercial
value. It suggests the space is cherished by the community for its accessibility and recreational use, not
for its potential financial gain. Hence, option (a) is not the correct answer.
(b) To emphasize its significance to the local community.
The term 'well-loved public space' is used to highlight the importance of Ontario Place to the local
community. It underscores the emotional attachment and the value placed on the space by residents,
making it clear why the redevelopment plan is controversial. Hence, option (b) is the correct answer.
(c) To suggest it is a place of historical battles.

Page 23 of 53
This interpretation does not align with the context of the passage. The term 'well-loved' does not
indicate historical battles but rather the current affection and significance of the space to the
community. The focus is on its present-day value rather than any historical conflicts. Hence, option (c)
is not the correct answer.
(d) To indicate it is rarely visited and underappreciated.
The term 'well-loved' suggests frequent visits and appreciation by the community, which is the opposite
of being rarely visited and underappreciated. The passage aims to convey that the space is highly valued
and regularly used by residents. Hence, option (d) is not the correct answer.

48. Correct Answer: (b) It establishes a concerning precedent that could lead to more public spaces
being privatized without thorough review.
Reference Lines: "The court’s recent decision 'sets a very terrible precedent for the future of our Ontario
public institutions.'"
Explanation:
(a) It sets a beneficial precedent ensuring strict environmental assessments for public sites.
This option is incorrect as the court's decision actually exempts the site from environmental
assessments, which does not ensure strict evaluations in the future. The passage indicates the opposite
impact, with less rigorous oversight. Hence, option (a) is not the correct answer.
(b) It establishes a concerning precedent that could lead to more public spaces being privatized without
thorough review.
This option accurately analyzes the impact of the court’s decision. The passage describes the decision
as setting a “terrible precedent,” suggesting that it could lead to the privatization of more public spaces
without adequate environmental and heritage reviews. Hence, option (b) is the correct answer.
(c) It has no significant impact on future decisions regarding public institutions.
This option is incorrect as the passage clearly states that the decision sets a precedent, implying it will
influence future actions. The described impact is significant, as it could affect the handling of other
public spaces and institutions. Hence, option (c) is not the correct answer.
(d) It ensures that heritage sites will receive more protection and thorough assessments in the future.
This option contradicts the passage, which indicates that the decision undermines protections for
heritage sites. The new law exempts Ontario Place from environmental assessment, reducing the level
of protection and oversight. Hence, option (d) is not the correct answer.

49. Correct Answer: (c) As an effort disregarding legal and environmental protocols for redevelopment.
Reference Lines: "Last December, one week after the request for judicial review was filed, the province
passed the Rebuilding Ontario Place Act, which nixed Ontario Place’s protections as a heritage site and
exempted it from environmental assessment."
Explanation:
(a) As a balanced approach respecting both development and heritage preservation.
The passage does not suggest a balanced approach. Instead, it describes actions that disregard heritage
preservation, such as bypassing environmental assessments and removing protections. The portrayal is
one-sided towards development without respect for preservation. Hence, option (a) is not the correct
answer.
(b) As a strategy primarily driven by public interest and environmental conservation.
This option is incorrect as the passage highlights the government's disregard for environmental
protocols and public input. The focus is on development rather than conservation or public interest.
Hence, option (b) is not the correct answer.
(c) As an effort disregarding legal and environmental protocols for redevelopment.
Page 24 of 53
The passage portrays the government’s actions as ignoring necessary legal and environmental
assessments. The description of passing a law to exempt the site from these evaluations supports this
interpretation, indicating a disregard for proper protocols. Hence, option (c) is the correct answer.
(d) As an initiative focusing on enhancing the site's historical and cultural significance.
This option is incorrect because the passage indicates the opposite. The government’s actions are
depicted as undermining the site's historical and cultural significance by removing heritage protections
and bypassing environmental assessments. Hence, option (d) is not the correct answer.

50. Correct Answer: (b) Ignoring the heritage status of Ontario Place.
Reference Lines: "the province passed the Rebuilding Ontario Place Act, which nixed Ontario Place’s
protections as a heritage site and exempted it from environmental assessment."
Explanation:
(a) Seeking community feedback before the redevelopment.
This option is incorrect as the passage indicates that the government did not seek community feedback.
The description of Ford collecting proposals without public input contradicts this interpretation. Hence,
option (a) is not the correct answer.
(b) Ignoring the heritage status of Ontario Place.
This option correctly identifies an example of the government's disregard for public input. The passage
describes the removal of heritage protections, which indicates a neglect of the site's historical
significance and public concern. Hence, option (b) is the correct answer.
(c) Including public opinion in the decision-making process.
This option is incorrect because the passage explicitly states that public input was not considered in the
redevelopment plans. The government’s actions are portrayed as unilateral without regard for public
opinion. Hence, option (c) is not the correct answer.
(d) Protecting environmental and heritage sites from redevelopment.
This option is incorrect as the passage describes the opposite action. The government removed
protections from Ontario Place, facilitating redevelopment without the necessary environmental and
heritage assessments. Hence, option (d) is not the correct answer.

51. Correct Answer: B. Indonesia

52. Correct Answer: B. Justin Trudeau

53. Correct Answer: A. Anuja

54. Correct Answer: B. Indian National Centre for Ocean Information Services (INCOIS)

55. Correct Answer: B. Anju Bobby George

56. Correct Answer: B. Jasprit Bumrah

57. Correct Answer: C. BHARATPOL

58. Correct Answer: C. Tamil Nadu

59. Correct Answer: B. Gujarat

60. Correct Answer: B. Mahatma Gandhi

Page 25 of 53
61. Correct Answer: A
Released by: - Institute for Economics and Peace (Sydney, Australia) * 2nd - Pakistan * 3rd - Syria * India's
rank - 14th

62. Answer: C.
Japan and India renew Bilateral Swap Arrangement for up to $75 billion. ✓ Japan and India have renewed
the Bilateral Swap Arrangement (BSA), effective from February 28, 2025, for up to $75 billion.

63. Answer: D.
‘Diyaslai’ is not just a book; it is a testament to an inspiring journey: Former President Ram Nath Kovind.
✓ The Indira Gandhi National Centre for the Arts (IGNCA), an autonomous institution under the Ministry
of Culture, in collaboration with Satyarthi Movement for Global Compassion, held a dedicated discussion
on Diyaslai, the autobiography of Nobel Peace Prize laureate Kailash Satyarthi.

64. Answer: D.
Major Dhyan Chand Khel Ratna Award (Highest Sports of India)
Gukesh D (Chess)
Harmanpreet Singh (Hockey)
Praveen Kumar (Para-Athletics)
Manu Bhaker (Shooting)

65. Answer: B. 139

66. Answer: B
General Elections 2024 {18th Lok Sabha}
Held From → 19 April to 1 June 2024 in 7 phases
To elect → 543 members of the Lok Sabha

67. Answer: C.
The Government nationalised 14 banks (with effect from the midnight of 19 July 1969) with deposits of
over Rs.50 crore by promulgating the Banking Companies (Acquisition and Transfer of Undertakings)
Ordinance, 1969.

68. Answer: C.

69. Answer: D
The Exercise Maitree is a combined field exercise between Thailand and India. The first exercise was held
in Sattahip, Chonburi, in 2006.

70. Answer: C Bhil


According to the 2011 census, the Bhils are the largest group in India, with a population of 4.61 million,
which is 37.7 % of total Scheduled Tribe population. Bhil tribes are spread over the states of Gujarat,
Madhya Pradesh Rajasthan, Maharashtra, Chhattisgarh and Andhra Pradesh. India's largest top three
tribal groups1) Bhil 37. 7 % 2) Gond 26. 2 % 3) Santhal 22.1

71. Correct Answer: A [Delhi]


The Delhi government approved the Mahila Samriddhi Yojana on International Women’s Day. It provides
Rs 2,500 per month to women aged 21-60 from weaker economic sections earning up to Rs 3 lakh
annually. The scheme aims to empower women with financial support and self-employment
opportunities. It has various forms, including monthly allowances, subsidized loans for micro-enterprises,
and vocational training. The scheme is designed to help women achieve stable incomes and contribute
to their families’ well-being.
Page 26 of 53
72. Correct Answer: C [Geneva, Switzerland]
The 353rd Governing Body meeting of the International Labour Organisation (ILO) is being held in Geneva,
Switzerland, from March 10 to 20, 2025. Representatives from governments, workers, and employers
discuss key labour and employment policies. India’s delegation, led by Sumita Dawra, Secretary, Ministry
of Labour and Employment, highlighted India’s labour welfare and employment efforts. India supported
the UN-led Second World Summit for Social Development in Doha, Qatar. India has doubled its social
protection coverage to 48.8%, exceeding the global average by 5%.

73. Correct Answer: A [Communication Satellite]


The Public Accounts Committee (PAC) raised concerns over the economic viability of GSAT-18 due to
underutilisation of six transponders until 2027. The committee recommended that the Department of
Space exercise greater financial prudence in satellite projects using public funds. A revenue loss of ₹117
crore was reported due to idle transponders. GSAT-18 is a communication satellite launched by ISRO on
October 5, 2016. It was launched using the Ariane-5 VA-231 rocket from Kourou, French Guiana, with a
liftoff weight of 3,404 kg. The satellite has a 15-year mission life (until 2032) and was injected into
Geosynchronous Transfer Orbit (GTO).

74. Correct Answer: A [Moscow]


The Russian capital was moved to Moscow in March 1918. Moscow is the capital and largest city of Russia.
On March 12, 1918 Moscow became the capital of the Soviet state.

75. Correct Answer: C [Odisha]


Bitar Kanika Mangrove Forest is located in the state of Odisha. It has an extent of 650 km in the deltas
of Brahmani and Baitarani River.

76. Correct Answer: C [36]


Lakshadweep is an archipelago consisting of 36 islands with an area of 32 sq km. A Union Territory, it is
comprised of 12 atolls, three reefs, five submerged banks and ten inhabited islands. The name
Lakshadweep in Malayalam and Sanskrit means ‘a hundred thousand Islands’.

77. Correct Answer: C [1991]


The USSR finally collapsed in 1991. On December 25, 1991, the Soviet hammer and sickle flag was
replaced by the Russian tricolor. Mikhail Gorbachev also resigned his post as President of the Soviet
Union.

78. Correct Answer: A [Lakshadweep]


Lakshadweep is the least populated Union Territory of India. It is an archipelago located in the Arabian
Sea with a total population of approximately 64,429 according to the latest Census. The main occupations
of people living here are fishing and coconut cultivation. It is considered an important strategic location
for India due to its isolated location.

79. Correct Answer: D [Rs. 50 Note]


The new 50 rupees banknote from India is in a bright turquoise colour. On the obverse side of the Rs.50
note is a portrait of Mahatma Gandhi, India’s independence movement leader. On the reverse side of the
₹50 bill is an image of the stone chariot in the UNESCO World Heritage Site of Hampi.

80. Correct Answer: A [Sri Shankar Kurup]


Sri Shankar Kurup was the first recipient of the Gyanpith award in 1965 for his extraordinary
achievements in Malayalam poetry.

81. Answer: (b) No sharks are cats.


Explanation:
Page 27 of 53
Premise 1: All cats are mammals.
Premise 2: Some mammals have fur.
Premise 3: No fish are mammals.
Premise 4: A shark is a fish.
From Premise 3 and Premise 4, since no fish are mammals and a shark is a fish, it follows that no sharks
are mammals. Given that all cats are mammals (Premise 1), no sharks can be cats, hence option (b) is
true.
Option (a) is not necessarily true as it does not follow directly from the premises.
Option (c) is not necessarily true because only some mammals have fur, not all.
Option (d) is false because no fish are mammals, so sharks cannot be mammals.

82. Answer: (a) Only I and II follow


Explanation:

Premise 1: Ethics is a branch of philosophy.


Premise 2: All branches of philosophy are theoretical disciplines.
Premise 3: Only theoretical disciplines can be academic subjects.
From Premise 1 and Premise 2, since ethics is a branch of philosophy and all branches of philosophy are
theoretical disciplines, it follows that ethics is a theoretical discipline (Statement I). From Premise 2 and
Premise 3, since ethics is a theoretical discipline, and only theoretical disciplines can be academic
subjects, ethics can be an academic subject (Statement II).
Statement III is incorrect because it contradicts Premise 2 and the conclusion derived from it.
Therefore, the most appropriate answer is (a) Only I and II follow.

83. Answer: (b) Only II follows


Explanation:

Page 28 of 53
Premise 1: All novels are works of fiction.
Premise 2: No works of fiction are scientific.
Premise 3: Only research papers are scientific.
From Premise 1 and Premise 2, since no works of fiction are scientific and novels are works of fiction, it
follows that novels are not scientific.
From Premise 3, only research papers are scientific, which means no works of fiction can be research
papers since works of fiction are not scientific.
Therefore, Statement II follows: no research papers are works of fiction.
Statement I is incorrect because it contradicts Premise 2 and 3.
Statement III is incorrect because it is impossible for any research papers to be works of fiction based
on Premise 2 and 3.
Thus, the most appropriate answer is (b) Only II follows.

84. Answer: (d) None of the above


Explanation:

Premise 1: Some teachers are writers.


Premise 2: All writers are artists.
Premise 3: No artist is a scientist.
Statement I is incorrect because no artist is a scientist, and all writers are artists.
Statement III is incorrect because if no artist is a scientist and some teachers are artists, then teachers
who are artists cannot be scientists.
Thus, the most appropriate answer is d

85. Answer: (b) Only II


Explanation:

Premise 1: Some fruits are apples.


Premise 2: All apples are red.
Page 29 of 53
Premise 3: Some red things are berries.
From Premise 1 and Premise 2, since all apples are red and some fruits are apples, it follows that some
fruits are red things. This makes Statement II true: Some red things are fruits.
Statement I is incorrect because the premises do not provide a direct link between berries and apples.
Statement III is incorrect because the premises do not establish a direct connection between berries
and fruits.
Thus, the most appropriate answer is (b) Only II.

86. Answer: (a) II and IV follow


Explanation:

Premise 1: All scientists are researchers.


Premise 2: Some researchers are teachers.
Premise 3: All teachers need degrees.
From the premises:
Statement I is incorrect because Y is a researcher but not necessarily a scientist; only some researchers
are scientists.
Statement II follows because some researchers are teachers, so Y may be a teacher.
Statement III is incorrect because Y is not necessarily a teacher; being a researcher does not imply being
a teacher.
Statement IV follows because if Y is a teacher, then according to Premise 3, Y needs a degree.
Thus, the most appropriate answer is (a) II and IV follow.

87. Answer: (a) No roses are machines.


Explanation:

Premise 1: All roses are flowers.


Premise 2: No flower is a car.
Premise 3: All cars are machines.
Premise 4: All machines are cars.
From Premise 1 and Premise 2, since all roses are flowers and no flower is a car, it follows that no roses
are cars.
Since Premise 3 and Premise 4 imply that cars and machines are interchangeable (all cars are machines
and all machines are cars), and no roses are cars, it follows that no roses can be machines.
Thus, the most appropriate answer is (a) No roses are machines.

Page 30 of 53
88. Answer: (b) At least some tools are machines
Explanation:

Premise 1: Some cars are vehicles.


Premise 2: All vehicles are machines.
Premise 3: Some machines are tools.
From Premise 1 and Premise 2, since all vehicles are machines, it follows that some cars are machines.
From Premise 3, since some machines are tools, it necessarily follows that at least some tools are
machines.
Thus, the most appropriate answer is (b) At least some tools are machines.

89. Answer: (b) If 'Y' is wild, it must be dangerous.


Explanation:

Premise 1: All birds are animals.


Premise 2: Some animals are wild.
Premise 3: Everything that is wild is dangerous.
From Premise 1 and Premise 2, we know that some animals (including birds) can be wild.
From Premise 3, we know that anything wild is dangerous.

90. Correct Answer:-b) ii and iii follow.


Explanation: - Based on the below diagram only, statement iii follows because as it given no
sweet thing is bitter, it logically follows that if Fruit is sweet, it will never be bitter.

91. Correct Answer: - (c) Only iii


Page 31 of 53
Explanation: - Based on the below diagram statement ii and iii follows.

92. Answer: (d) Tiger is a pet. As to whether it is a bird, and as to whether it flies, there is not sufficient
information.
Explanation:

Premise 1: All bird’s fly.


Premise 2: Some birds are pets.
Premise 3: All cats are pets.
Given: Tiger is a cat.
From Premise 3 and the given information, we know that Tiger is a pet.
Statement (a) is incorrect because there is no information indicating that Tiger flies.
Statement (b) is incorrect because there is no information indicating that Tiger is a bird.
Statement (c) is incorrect because while Tiger is a pet, there is insufficient information to conclude
whether it flies or is a bird.
Statement (d) is correct because Tiger is a pet, but there is not enough information to determine if it is
a bird or if it flies.
Thus, the most appropriate answer is (d) Tiger is a pet. As to whether it is a bird, and as to whether it
flies, there is not sufficient information.

93. Answer: (a) X


Explanation: -
Based on the given instructions we can deduce the following
From this, the order from longest to shortest time is:
Y>W>V>Z>X
Page 32 of 53
From this, the order from most to least discoveries is:
W>Y>Z>V>X
From this, the order from most to least papers is:
X>Z>W>V>Y

94. Answer: (c) W


Explanation: - Its directly given in the passage

95. Answer: (b) Y


Explanation: -
From this, the order from longest to shortest time is:
Y>W>V>Z>X

96. Answer: (d) V


Explanation: -
From this, the order from most to least papers is:
X>Z>W>V>Y

97. Answer: (c) X


Explanation: - From this, the order from most to least discoveries is:
W>Y>Z>V>X

98. Answer: (b) Star


Explanation: A molecule is composed of atoms, and similarly, a galaxy is composed of stars. Therefore,
the relationship is that of a whole to its fundamental components.

99. Answer: (a) Car


Explanation: Mitochondria are the powerhouses of the cell, providing energy for its functions, much like
an engine provides the power for a car to operate.

100. Answer: (c) Respiration


Explanation: A leaf performs photosynthesis for a plant, converting light into energy, while a lung
performs respiration for an organism, exchanging gases to provide oxygen to the body.

101. Answer: (d) Meal


Explanation: Grammar is to a sentence as ingredients are to a meal. Grammar rules dictate the structure
of a sentence, while ingredients are the essential components used to create a meal.

102. Correct Answer: - (b)


Explanation: - 6 is the product of 2 and 3, 18 is the product of 6 and 3 similarly the next number will be
the product of 108 and 18

103. Answer: (b) 49


Explanation: This series consists of the squares of consecutive natural numbers:
1^2 =1
2^2 =4
3^2 =9
Page 33 of 53
4^2 =16
5^2=25
6^2=36
Following this pattern, the next number is 7^2 =49
Therefore, the next number in the series is 49.

104. Answer: (d) S


Explanation: This series involves increasing the position in the alphabet by an increasing number:
A (1st letter of the alphabet)
D (A + 3 = 4th letter)
H (D + 4 = 8th letter)
M (H + 5 = 13th letter)
Following this pattern, the next letter is:
M + 6 = 19th letter (S)

105. Answer: (c) 160


Explanation: This series involves doubling each previous number:
5 × 2 = 10
10 × 2 = 20
20 × 2 = 40
40 × 2 = 80
Following this pattern, the next number is:
80 × 2 = 160
Therefore, the next number in the series is 160.

106. Correct Answer: (a) Nephew


Explanation:
Let's break down the relationships step by step:
A is the son of (b)
B is married to C, so C is A's mother.
C has a daughter named D, making D A's sister.
D is married to E, so E is A's brother-in-law.
E and D have a son named F, making F D's son.
Since D is A's sister, F, who is D's son, would be A's nephew. Therefore, F is A's nephew.

107. Correct Answer: (b) Grandmother


Explanation: Let's break down the relationships step by step:
P is the father of Q.
Q is the mother of R.
S is the wife of P, making S Q's mother.
Since Q is the mother of R and S is Q's mother, S is the grandmother of R. Therefore, S is R's grandmother.

108. Correct Answer: (c) Father-in-law


Anil states that the gentleman's wife is the mother-in-law of his daughter. This relationship indicates
that the gentleman's wife is the mother of Anil's daughter's husband. Thus, the gentleman himself is the
father of Anil's son-in-law. In family relational terms, this makes the gentleman the father-in-law to
Anil's daughter. Therefore, the correct answer is: Option (c) Father-in-law
Page 34 of 53
109. Correct Answer: (b) Cousin
Explanation:
Nina explains that the person in the photo is the granddaughter of her grandfather's brother. This means
the person's grandparent (her grandfather's brother) and Nina's grandfather are siblings. Thus, the
person’s parent (child of the brother) and Nina are first cousins, making the person in the photograph
Nina’s first cousin once removed. However, from Nina's perspective, the simplest familial term is
"cousin," as it generally encompasses various levels of removed cousin relationships in colloquial use.
Therefore, option (b) Cousin is the correct choice.

110. Correct Answer: (b) Mother


Explanation: Maya states that the man's mother is the only daughter of her (Maya's) mother. This means
that Maya herself is the only daughter of her mother. Consequently, if the man's mother is the only
daughter of Maya's mother, the man's mother must be Maya herself. Therefore, Maya is the mother of
the man. Thus, the correct answer is option (b) Mother.

111. Correct Answer: (b) Individuals fleeing from conflict zones often experience multiple layers of
cultural adjustment upon resettlement.
Explanation:
Hamza grew up in a Syrian village on the Turkish border, where her father worked as a greengrocer. In
2012, as opposition protests turned to vicious civil war, the family fled Syria for Lebanon and a life of
hardship lived on the margins of society. Eventually, they were offered a place on a UN refugee
resettlement programme and in 2018 arrived here in the very west of Wales, where the land runs out.
It was, she has said, a disconcerting experience of layered culture shocks.
Which of the following assumptions is made in the passage?
(a) Refugee resettlement programs are aimed primarily at providing immediate safety rather than long-
term stability.
This option suggests that the primary goal of refugee resettlement programs is to ensure immediate
safety for refugees. However, the passage does not focus on the objectives of the resettlement programs.
Instead, it highlights Hamza's experience of cultural shocks upon arriving in Wales. Hence, Option (a) is
not the correct answer.
(b) Individuals fleeing from conflict zones often experience multiple layers of cultural adjustment upon
resettlement.
This is the correct answer because the passage emphasizes Hamza's experience of "layered culture
shocks," which implies that such experiences are common among refugees resettling in new countries.
It assumes that individuals fleeing conflict zones will face multiple layers of cultural adjustment. Hence,
Option (b) is the correct answer.
(c) Economic hardship in host countries is primarily due to the inability of refugees to integrate into the
local economy.
This option suggests that economic hardship in host countries is caused by refugees' inability to
integrate economically. However, the passage does not discuss economic conditions in the host country
or the refugees' integration into the local economy. The focus is on cultural adjustment rather than
economic factors. Hence, Option (c) is not the correct answer.
(d) Western countries are generally unprepared for the cultural challenges posed by incoming refugees.
This option implies that Western countries are unprepared for the cultural challenges brought by
refugees. While the passage describes Hamza's cultural shock, it does not make a broad assumption

Page 35 of 53
about the preparedness of Western countries for such challenges. Hence, Option (d) is not the correct
answer.

112. Correct Answer: (d) The initial surge of support for racial equality movements was primarily driven
by superficial or performative actions.
Explanation:
(a) Public interest in social justice movements is highly susceptible to fluctuations over time.
This option is plausible as it aligns with the idea that movements can lose momentum over time.
However, the passage specifically attributes the decline of Black Lives Matter to the superficiality of
initial support and unfulfilled promises, rather than general fluctuations in public interest. Hence,
Option (a) is not the correct answer.
(b) Political leaders have consistently failed to implement meaningful reforms in response to social
movements.
This option implies that the decline of the movement is due to political leaders' failure to enact reforms.
While the passage reflects disappointment with outcomes, it does not specifically blame political
leaders. The focus is more on the performative nature of initial support rather than political inaction.
Hence, Option (b) is not the correct answer.
(c) Economic challenges have diverted attention from social issues, undermining their progress.
This option suggests that economic challenges have shifted focus away from social issues, but the
passage centers on the notion that the promises were never deeply rooted. The decline is attributed
more to the superficiality and lack of genuine pursuit rather than economic distractions. Hence, Option
(c) is not the correct answer.
(d) The initial surge of support for racial equality movements was primarily driven by superficial or
performative actions.
This option correctly identifies the assumption that underlies the author’s argument. The passage
suggests that the diversity efforts were more about appearance than substantive change, and these
efforts were discarded once the immediate fervor passed. Hence, Option (d) is the correct answer.

113. Correct Answer: (a) The mediums through which the younger generation engages with reading are
as effective in fostering a love for literature as traditional books.
Explanation:
(a) The mediums through which the younger generation engages with reading are as effective in
fostering a love for literature as traditional books.
This option is the correct answer as it addresses the core assumption that underpins the passage's
conclusion. For the joy of reading to not be considered lost, the new mediums must be seen as equally
capable of instilling a love for literature, validating the conclusion that children still enjoy reading
through their preferred formats. Hence, Option (a) is the correct answer.
(b) The younger generation spends an equal amount of time reading as the previous generations did,
albeit through different formats.
While this option seems relevant, it does not support the conclusion about the joy of reading. The time
spent reading does not necessarily equate to the joy or effectiveness of reading. Hence, Option (b) is not
the correct answer.
(c) Parents' nostalgia for their own childhood reading experiences prevents them from recognizing new
forms of literary engagement.
This option explains the parents' perspective but does not directly support the conclusion that the joy
of reading is still present in the younger generation. It focuses on the parents' viewpoint rather than the
effectiveness of the new reading mediums. Hence, Option (c) is not the correct answer.
Page 36 of 53
(d) The shift from traditional reading to digital mediums has not negatively impacted the overall
intellectual development of the younger generation.
Although this option touches on a potential concern related to the shift in reading habits, it does not
directly support the conclusion about the joy of reading. Intellectual development is a broader concept
and does not specifically address the joy or engagement with reading. Hence, Option (d) is not the
correct answer.

114. Correct Answer: (a) Recent satellite images show significant construction activity at the Ream
Naval Base, including new piers and barracks.
Explanation
(a) Recent satellite images show significant construction activity at the Ream Naval Base, including new
piers and barracks.
This option strengthens the concern because it provides concrete evidence of military infrastructure
development at the Ream Naval Base. This construction activity supports the idea that the base could
be used for military purposes, aligning with concerns about it becoming an outpost for the Chinese navy.
Hence, Option (a) is the correct answer.
(b) The Cambodian government has announced a new policy to restrict foreign military presence in the
country.
This option would actually weaken the concern as it suggests that Cambodia is taking steps to prevent
foreign military establishments, including potentially a Chinese naval outpost. Therefore, it does not
strengthen the argument. Hence, Option (b) is not the correct answer.
(c) Historical records indicate that Sihanoukville Port has primarily been used for commercial shipping
rather than military purposes.
This option does not address the current or future use of the Ream Naval Base. It provides historical
context about the port but does not contribute to the argument that the base might be used for military
purposes. Hence, Option (c) is not the correct answer.
(d) Reports reveal that local Cambodian communities are protesting the increased military activity in
the region.
While this option indicates local discontent with military activities, it does not provide specific evidence
that the Ream Naval Base is being developed into a Chinese naval outpost. It highlights local reactions
rather than strategic military developments. Hence, Option (d) is not the correct answer.

115. Correct Answer: (a) Strict regulatory norms ensure that NBFCs maintain a high level of financial
stability and protect the interests of borrowers.
Explanation:
(a) Strict regulatory norms ensure that NBFCs maintain a high level of financial stability and protect the
interests of borrowers.
This option is the correct answer as it directly justifies the RBI's increased scrutiny. By emphasizing the
importance of regulatory norms in maintaining financial stability and protecting borrowers, it aligns
with the RBI's rationale for tightening its grip on NBFCs and addressing any deviations from established
guidelines. Hence, Option (a) is the correct answer.
(b) The RBI has found that NBFCs are the primary source of credit for small and medium-sized
enterprises (SMEs) in India.
While this statement highlights the significance of NBFCs in providing credit to SMEs, it does not directly
justify the RBI's increased scrutiny of lending norms. It merely explains the role of NBFCs rather than
the need for regulatory oversight. Hence, Option (b) is not the correct answer.
(c) Several NBFCs have reported a significant increase in their lending activities over the past year.
Page 37 of 53
This option indicates a rise in lending activities by NBFCs but does not address why increased scrutiny
or strict adherence to regulatory norms is necessary. The increase in lending activity alone does not
justify regulatory actions. Hence, Option (c) is not the correct answer.
(d) NBFCs play a crucial role in the Indian financial system by providing credit to sectors underserved
by traditional banks.
This option acknowledges the importance of NBFCs in the financial system but does not directly relate
to the need for strict regulatory norms or increased scrutiny by the RBI. It focuses on the role of NBFCs
rather than the rationale behind regulatory measures. Hence, Option (d) is not the correct answer.

116. Correct Answer: (d) The accessibility and convenience of electronic media surpass those of print
media for most individuals.
Explanation:
(a) The decline in print media circulation is due to its inability to adapt to modern technological
advancements.
This option focuses on a possible cause for the decline in print media, attributing it to its failure to adapt
to technological changes. However, it does not address the central question of the utility of print media
compared to electronic media in the current digital age. The argument about print media's utility being
questionable depends on a comparison of current utility rather than historical adaptation issues. Hence,
option (a) is not the correct answer.
(b) Print media is unable to provide the same level of interactivity and instant updates that electronic
media offers.
This statement highlights a disadvantage of print media by noting its lack of interactivity and instant
updates compared to electronic media. While this is a relevant point, it does not directly address the
overall utility comparison between print and electronic media in today's age. The assumption in
question needs to cover broader accessibility and convenience issues rather than specific features like
interactivity. Hence, option (b) is not the correct answer.
(c) The environmental impact of producing physical publications is a significant concern for
contemporary consumers.
This option brings in an environmental perspective, suggesting that the production of print media might
be less favorable due to its environmental impact. While this could be a factor influencing the utility of
print media, the argument primarily revolves around convenience and accessibility issues in the digital
age, rather than environmental concerns. Hence, option (c) is not the correct answer.
(d) The accessibility and convenience of electronic media surpass those of print media for most
individuals.
This option supports the argument by addressing the critical factors of accessibility and convenience,
which are fundamental to the comparison between print and electronic media in today's digital age. It
assumes that electronic media's superior accessibility and convenience make it more useful and
preferable, thus making the utility of print media questionable. Hence, option (d) is the correct answer.

117. Correct Answer: (c) Introducing energy efficiency norms for additional products is a strategy to
reduce overall energy consumption.
Explanation:
(a) Consumers are expected to prefer appliances with energy efficiency labels due to their cost-saving
potential.
This option suggests a likely outcome of the BEE's initiative, implying that consumers might favor
energy-efficient appliances for their cost-saving benefits. However, the passage does not provide
specific evidence that consumers' preferences are the primary focus. The passage emphasizes providing
Page 38 of 53
information for informed decisions, not predicting consumer behavior. Hence, option (a) is not the
correct answer.
(b) The BEE’s voluntary labelling scheme aims to make manufacturers more competitive in the market.
This option suggests that the labelling scheme is designed to increase market competitiveness among
manufacturers. While this could be an indirect effect, the passage does not indicate that the primary aim
of the scheme is to boost competition. Instead, it focuses on consumer information and energy efficiency.
Hence, option (b) is not the correct answer.
(c) Introducing energy efficiency norms for additional products is a strategy to reduce overall energy
consumption.
This option aligns with the implicit goal of the BEE's initiative. By setting energy efficiency norms and
promoting labelling, the BEE aims to encourage the use of energy-efficient products, which would
naturally lead to reduced energy consumption. This inference fits well with the overall intent described
in the passage. Hence, option (c) is the correct answer.
(d) The BEE plans to mandate energy efficiency norms for all household appliances in the future.
This option implies that the BEE has a plan to make energy efficiency norms mandatory for all
household appliances. However, the passage only mentions a voluntary labelling scheme and does not
indicate any future plans for mandatory norms. Hence, option (d) is not the correct answer.

118. Correct Answer: (a) Sustainable energy development initiatives that do not consider gender
equality are likely to be less effective.
Explanation:
(a) Sustainable energy development initiatives that do not consider gender equality are likely to be less
effective.
This option correctly concludes that ignoring gender equality would likely render sustainable energy
initiatives less effective. The passage stresses the fundamental role of gender equality in achieving
sustainable energy for all, suggesting that any initiative not considering this aspect would be incomplete
and, therefore, less effective. Hence, option (a) is the correct answer.
(b) The achievement of gender equality alone is sufficient to achieve all Sustainable Development Goals
(SDGs).
This statement misinterprets the passage, which asserts that gender equality is fundamental but not the
sole factor in achieving SDGs. Many other elements are also critical for achieving these goals, and gender
equality alone is not sufficient. Hence, option (b) is not the correct answer.
(c) Conversations around gender equality should prioritize sustainable energy development over other
issues.
While the passage emphasizes the importance of the intersection between gender equality and
sustainable energy, it does not suggest prioritizing this over other issues. The passage highlights one
overlooked issue among many, not suggesting an overall prioritization strategy. Hence, option (c) is not
the correct answer.
(d) Gender equality initiatives have historically been separate from sustainable energy development
efforts.
The passage does not provide any historical context or imply that gender equality initiatives have been
separate from sustainable energy efforts. It focuses on the importance of their intersection moving
forward, not on past efforts. Hence, option (d) is not the correct answer.

119. Correct Answer: (c) Both I and II follow


Explanation:
(a) Only I follows
Page 39 of 53
This option implies that only the first conclusion is supported by the passage. The passage does indicate
that "net zero" gained significant attention post-2021 Glasgow climate summit. However, it also
discusses new legislation and governmental actions towards clean energy, supporting both conclusions.
Hence, option (a) is not the correct answer.
(b) Only II follows
This option implies that only the second conclusion follows from the passage. The passage indeed
discusses new laws indicating a governmental push for clean energy, but it also supports the spread of
attention on "net zero" since the Glasgow summit. Hence, option (b) is not the correct answer.
(c) Both I and II follow
This option is correct as the passage provides evidence for both conclusions. The passage confirms that
"net zero" gained international attention following the Glasgow climate summit and highlights new laws
showing governmental efforts towards clean energy, supporting both conclusions. Hence, option (c) is
the correct answer.
(d) Neither I nor II follows
This option is incorrect because the passage clearly supports both conclusions. It discusses the rise in
attention to "net zero" since the Glasgow summit and mentions legislative actions indicating a push for
clean energy. Hence, option (d) is not the correct answer.

120. Correct Answer: (d) Polling data shows that voters are more concerned with economic issues than
with polarising social rhetoric.
Explanation:
(a) Surveys show that public support for BJP has declined in regions targeted by divisive rhetoric.
This option provides evidence of a decline in support specifically in areas targeted by divisive rhetoric.
While relevant, it does not address the broader argument about the overall failure of polarisation
attempts. It focuses on a specific effect rather than the overall outcome. Hence, option (a) is not the
correct answer.
(b) Reports indicate an increase in political violence in areas where polarising arguments were made.
This option suggests that polarising arguments led to an increase in violence, indicating significant
impact rather than failure. The argument is about the unsuccessful attempts at polarisation, not the
consequences of such attempts. Hence, option (b) is not the correct answer.
(c) Media coverage of BJP's divisive arguments has focused mainly on their potential to create social
unrest.
This option highlights media focus on the potential for social unrest due to divisive arguments. While
this points to negative coverage, it does not directly support the argument that the polarisation attempts
were unsuccessful. It discusses media reactions rather than the effectiveness of the attempts. Hence,
option (c) is not the correct answer.
(d) Polling data shows that voters are more concerned with economic issues than with polarising social
rhetoric.
This option supports the passage's claim that the polarisation attempts were unsuccessful. It shows that
voters prioritize economic issues over divisive social rhetoric, suggesting that the efforts to polarise did
not resonate with the electorate. This aligns with the argument that other significant concerns
overshadowed the polarising efforts. Hence, option (d) is the correct answer.

121. Correct Answer: (b) Schemas allow individuals to organize and retrieve information efficiently.
Explanation:
(a) Memories can be altered by external influences such as new information or events.

Page 40 of 53
This option suggests that memories are not static and can be changed by new information or events.
While this may be true, the passage's main argument is about how underlying beliefs and cultural
assumptions form schemas that organize memories, not the alteration of memories themselves. The
author's focus is on the influences that shape memory content, not on the modification of existing
memories. Hence, option (a) is not the correct answer.
(b) Schemas allow individuals to organize and retrieve information efficiently.
This option is central to the author's argument. The passage describes schemas as mental frameworks
that help organize information based on perceived relationships and associations. For the argument to
hold, it must be assumed that these schemas function efficiently in organizing and retrieving
information. Without this assumption, the argument about the role and influence of schemas on
memory content would fall apart. Hence, option (b) is the correct answer.
(c) The content of a person's memory is solely determined by their direct experiences.
This option contradicts the passage's argument, which states that memories are influenced by a
combination of personal beliefs, social pressures, biases, heuristics, and cultural assumptions. The
passage explicitly argues against the idea that memory content is solely based on direct experiences.
Therefore, assuming this would undermine the argument. Hence, option (c) is not the correct answer.
(d) Cultural assumptions have a negligible effect on the formation of schemas.
This option opposes the passage's content. The passage emphasizes that cultural assumptions are one
of the significant factors that influence the formation of schemas. Assuming that cultural assumptions
have a negligible effect would contradict the passage's argument about the various influences on
memory content. Hence, option (d) is not the correct answer.

122. Correct Answer: (a) The consumption patterns of rural and urban populations are significantly
different, and the CPI does not account for these variations.
Explanation:
(a) The consumption patterns of rural and urban populations are significantly different, and the CPI
does not account for these variations.
This option most weakens the argument that the CPI accurately reflects the economy's cost of living. If
the CPI does not account for the different consumption patterns between rural and urban populations,
it cannot accurately measure the cost of living for the entire population, thus undermining its validity
as a comprehensive economic indicator. Hence, option (a) is the correct answer.
(b) The CPI basket includes outdated items that no longer represent the current consumption habits of
the majority of the population.
While this would affect the accuracy of the CPI, the passage does not provide information about the
timeliness of the items in the CPI basket. Therefore, this option does not directly address the accuracy
issue discussed in the passage. Outdated items could misrepresent current consumption patterns, but
the passage does not make it clear if this is a significant issue. Hence, option (b) is not the correct answer.
(c) Vegetables, despite their weight in the CPI basket, have shown highly volatile prices which distort
the overall inflation measurement.
Although price volatility can affect the CPI, the passage indicates that vegetables have a relatively small
weight (6.04%) in the CPI basket. This suggests that their volatility alone would not significantly distort
the overall inflation measurement. The argument about the accuracy of the CPI considers the overall
basket, not just one component. Hence, option (c) is not the correct answer.
(d) The methodology used to calculate the CPI has been consistent over the years, ensuring stability in
tracking inflation trends.
This option actually strengthens the argument by suggesting that the CPI's consistent methodology
ensures stability in tracking inflation trends, rather than weakening it. Consistency in methodology
Page 41 of 53
helps maintain accuracy over time, making this an argument for, rather than against, the CPI's accuracy.
Hence, option (d) is not the correct answer.

123. Correct Answer: (b) Promoting the joy and benefits of participating in amateur sports regardless
of the level of success in professional arenas.
Explanation:
(a) Encouraging young athletes to focus solely on becoming professional players and investing all their
resources in achieving that goal.
This action goes against the author's perspective, which celebrates the value and joy of amateur sports
rather than solely focusing on professional success. The passage emphasizes that being an amateur is
wonderful and should be appreciated, not seen as a lesser pursuit compared to professional sports.
Hence, option (a) is not the correct answer.
(b) Promoting the joy and benefits of participating in amateur sports regardless of the level of success
in professional arenas.
This is the correct action as it aligns perfectly with the author's argument that being an amateur athlete
is wonderful and should be celebrated for its own merits, not just in comparison to professional sports.
The passage emphasizes the intrinsic value and joy of amateur sports, making this the most aligned
action. Hence, option (b) is the correct answer.
(c) Criticizing amateur athletes for not being as dedicated or skilled as their professional counterparts.
This action contradicts the author's viewpoint by implying that amateur athletes are lesser and should
be criticized, which the passage explicitly argues against. The author views amateur sports as valuable
and enjoyable, not as something to be criticized for lacking professionalism. Hence, option (c) is not the
correct answer.
(d) Establishing a clear distinction between amateur and professional sports to emphasize the
superiority of the latter.
This option goes against the author's argument, which suggests that amateur sports should not be
viewed as lesser compared to professional sports, but rather appreciated for their own unique value.
The author argues against the notion of amateur sports being "lesser" and promotes the idea that they
are a gift. Hence, option (d) is not the correct answer.

124. Correct Answer: (b) The traditional grazing practices are no longer viable due to changing
environmental conditions.
Explanation:
(a) Agroforestry is the most effective solution for all regions affected by climate change.
This option is not necessarily taken for granted in the passage. While agroforestry is mentioned as a
potential solution, the passage does not imply that it is the most effective for all regions. The focus is on
the specific challenges faced by high-altitude pastoralists, not on the universal effectiveness of
agroforestry. Hence, option (a) is not the correct answer.
(b) The traditional grazing practices are no longer viable due to changing environmental conditions.
This is the correct assumption. The passage highlights the difficulties faced by pastoralists due to
climate change, such as shorter grazing seasons and less pasture for yaks, implying that traditional
grazing practices are no longer viable. The argument assumes that these traditional practices cannot
sustain pastoralists under current climate conditions. Hence, option (b) is the correct answer.
(c) Climate change impacts all agricultural practices equally, irrespective of the region.
This option is not supported by the passage. The passage specifically discusses the unique challenges
faced by high-altitude pastoralists, indicating that the impact of climate change can vary by region and

Page 42 of 53
practice. It does not assume uniform impact across all agricultural practices. Hence, option (c) is not the
correct answer.
(d) Pastoralists are unwilling to adapt to new methods that could mitigate the effects of land
degradation.
This option is not taken for granted in the passage. On the contrary, the passage mentions that
pastoralists like Dolma are trying to adapt through practices such as agroforestry, even though it is
challenging due to the altitude. The assumption is that adaptation is necessary, not that pastoralists are
unwilling to adapt. Hence, option (d) is not the correct answer.

125. Correct Answer: (d) Homemade fertilizers made from food waste can be as effective as commercial
fertilizers.
Explanation:
(a) Chemical fertilizers are harmful to the environment and should be avoided.
This option is not directly addressed in the passage. The passage focuses on the benefits of banana peels
as a fertilizer rather than the drawbacks of chemical fertilizers. Therefore, it is not an assumption made
in the passage. The discussion centers on the positive aspects of using banana peels, not on avoiding
chemical fertilizers. Hence, option (a) is not the correct answer.
(b) The nutrients found in banana peels are essential for the growth and health of all plants.
While the passage mentions that banana peels contain important nutrients, it does not imply that these
nutrients are essential for the growth and health of all plants. The passage focuses on the benefits these
nutrients provide, not their absolute necessity for all plants. It highlights the potential of banana peels
as a fertilizer, not a requirement. Hence, option (b) is not the correct answer.
(c) Sustainable gardening practices are more effective than traditional gardening methods.
This option is not supported by the passage. The passage discusses a specific sustainable practice but
does not compare the overall effectiveness of sustainable gardening practices to traditional methods.
The focus is on the benefits of banana peels, not on a general comparison of gardening practices. Hence,
option (c) is not the correct answer.
(d) Homemade fertilizers made from food waste can be as effective as commercial fertilizers.
This is the correct assumption. The passage suggests that using banana peels, a form of food waste, as
fertilizer can significantly benefit plants by providing essential nutrients, implying that homemade
fertilizers can be as effective as commercial options. The emphasis is on the potential effectiveness of
banana peel fertilizer, indicating its comparability to commercial fertilizers. Hence, option (d) is the
correct answer.

126. Correct Answer: (b) Homemade fertilizers made from food waste can be as effective as commercial
fertilizers.
Explanation:
(a) Chemical fertilizers are harmful to the environment and should be avoided.
This option is irrelevant to the passage. The passage discusses the concept of meaning and its subjective
and objective aspects. There is no mention of chemical fertilizers or their environmental impact.
Therefore, assuming this statement is not related to the given text. Hence, Option (a) is not the correct
answer.
(b) Homemade fertilizers made from food waste can be as effective as commercial fertilizers.
This assumption fits the passage, suggesting that banana peels, a form of food waste, can be used as an
effective fertilizer. The implication is that homemade solutions can provide similar benefits to
commercial products, emphasizing the practical and beneficial use of everyday items. Hence, Option (b)
is the correct answer.
Page 43 of 53
(c) The nutrients found in banana peels are essential for the growth and health of all plants.
The passage mentions the importance of meaning in subjective experiences and definitions. It does not
discuss the necessity of nutrients from banana peels for plant health. The scope of the passage is too
broad to support this specific agricultural claim. Hence, Option (c) is not the correct answer.
(d) Sustainable gardening practices are more effective than traditional gardening methods.
The passage does not compare sustainable and traditional gardening methods. It focuses on the concept
of meaning in a broad sense, not delving into specific agricultural practices or their effectiveness.
Therefore, this assumption is outside the scope of the passage. Hence, Option (d) is not the correct
answer.

127. Correct Answer: (a) The Chabahar Port provides India with a strategic alternative to bypass
Pakistan for trade with Afghanistan and Central Asia.
Explanation:
(a) The Chabahar Port provides India with a strategic alternative to bypass Pakistan for trade with
Afghanistan and Central Asia.
This option supports India's decision by highlighting the strategic advantage of the Chabahar Port.
Bypassing Pakistan allows India to secure a more stable and direct route for trade with Afghanistan and
Central Asia, which can be crucial given the geopolitical tensions. Hence, Option (a) is the correct answer.
(b) The domestic political instability in Iran due to the helicopter accident could lead to challenges in
honoring international contracts.
This option highlights a potential risk rather than supporting the decision. Political instability in Iran
could indeed pose challenges to the smooth execution of international agreements, making it an
argument against the contract. Hence, Option (b) is not the correct answer.
(c) The ongoing conflict in Gaza and heightened Israel-Iran tensions could disrupt operations at
Chabahar Port.
This option introduces a risk factor that could negatively impact the port's operations. It does not
provide a supporting argument for the contract but instead points out the potential for geopolitical
conflicts to affect the port's functionality. Hence, Option (c) is not the correct answer.
(d) Similar port investments by India in other regions have resulted in financial losses due to unforeseen
geopolitical tensions.
This option argues against the decision by pointing to a history of financial losses in similar investments.
It suggests that geopolitical tensions can negatively impact the success of such projects, thus not
supporting the current contract. Hence, Option (d) is not the correct answer.

128. Correct Answer: (c) Local communities have benefited economically from government incentives
to adopt sustainable livestock farming practices.
Explanation:
(a) The state has implemented a robust system to rehabilitate abandoned cattle, ensuring they do not
impact farmland.
This option suggests a mitigation strategy for the issue of abandoned cattle. However, it does not
inherently argue in favor of the state's policies as it implies a reactionary measure rather than a
proactive benefit stemming from the original policies. Hence, Option (a) is not the correct answer.
(b) The number of cattle-related accidents on highways has significantly decreased due to stricter
livestock management policies.
This option addresses a different issue, focusing on highway accidents rather than the core problem of
abandoned cattle affecting farmland and livelihoods. Therefore, it does not directly support the state's

Page 44 of 53
policies regarding livestock management in the context provided. Hence, Option (b) is not the correct
answer.
(c) Local communities have benefited economically from government incentives to adopt sustainable
livestock farming practices.
This option provides a direct positive outcome linked to the state's policies. It indicates that the policies
have resulted in economic benefits for local communities, which supports the argument in favor of the
state's approach to livestock management. Hence, Option (c) is the correct answer.
(d) There has been a marked increase in agricultural productivity in regions where abandoned cattle
have been a problem.
This option is contradictory to the passage, which suggests that abandoned cattle are decimating
harvests. An increase in productivity in such regions would likely not support the argument for the
state's current policies, which have led to the cattle problem. Hence, Option (d) is not the correct answer.

129. Correct Answer: (a) Studies show that women who eat last at the table consume fewer calories and
nutrients than they need for optimal health.
Explanation:
(a) Studies show that women who eat last at the table consume fewer calories and nutrients than they
need for optimal health.
This option supports the argument by providing evidence that the practice of eating last results in
women consuming insufficient calories and nutrients, which can harm their health. It clearly links the
practice of "food patriarchy" to negative health outcomes. Hence, Option (a) is the correct answer.
(b) Women in households where "food patriarchy" is practiced have higher rates of nutritional
deficiencies compared to men.
While this option suggests a negative impact on women's health, it does not directly support the
argument by explaining how "food patriarchy" specifically causes these deficiencies. It implies a
correlation but lacks a causal explanation tied to the practice itself. Hence, Option (b) is not the correct
answer.
(c) In households where "food patriarchy" is not practiced, there is no significant difference in health
outcomes between men and women.
This option implies equality in health outcomes but does not directly link "food patriarchy" to negative
health effects on women. It suggests a lack of disparity rather than explaining the detrimental impact of
the practice. Hence, Option (c) is not the correct answer.
(d) In families where gender equality is emphasized in education, women tend to achieve higher levels
of academic and professional success.
While this option supports the importance of gender equality in education, it does not address the
impact of "food patriarchy" on women's health. It is focused on educational and professional success
rather than health outcomes related to eating practices. Hence, Option (d) is not the correct answer.

130. Correct Answer: (a) The installation of public charging stations in urban areas has increased by
50% in the past year.
Explanation:
(a) The installation of public charging stations in urban areas has increased by 50% in the past year.
This option addresses the concern of charging inconvenience by suggesting that there are now
significantly more public charging options available. The increase in public charging stations reduces
the reliance on having consistent off-street parking, thereby undermining the argument that charging
an EV is significantly inconvenient. Hence, Option (a) is the correct answer.
(b) The majority of EV owners report satisfaction with their vehicle’s range and charging speed.
Page 45 of 53
While this indicates overall satisfaction with EVs, it does not specifically address the issue of charging
convenience for those without access to consistent off-street parking. Satisfaction with range and
charging speed does not negate the inconvenience faced by a subset of potential buyers. Hence, Option
(b) is not the correct answer.
(c) Some companies are developing portable charging solutions that can be used without dedicated
parking spaces.
This option suggests a potential future solution but does not undermine the current argument about
the inconvenience of charging for many potential buyers. The development of portable solutions is
promising but does not address the existing situation. Hence, Option (c) is not the correct answer.
(d) Advances in battery technology are expected to increase the driving range of EVs significantly in the
next five years.
This option speaks to future improvements in battery technology but does not directly address the
current inconvenience of charging EVs, especially for those without off-street parking. Future
advancements do not alleviate present concerns for potential buyers. Hence, Option (d) is not the
correct answer.

131. Correct Answer: (c) Both I and IV


Explanation:
(a) Only I
Conclusion I is directly supported by the passage, which states that "accidents of life" have become more
important in determining an individual's socioeconomic status in the 21st century. This reflects a shift
from inherited socioeconomic status to individual circumstances and life events as the primary
determinants of poverty. Hence, Option (a) is not the correct answer.
(b) Only I and III
Conclusion III is implicitly supported by the passage because the shift in poverty determinants from
birth-related factors to life events suggests underlying structural changes in society. However, the
passage does not explicitly discuss these structural changes in detail. Hence, Option (b) is not the correct
answer.
(c) Both I and IV
Conclusion IV is supported by the passage, as it implies that addressing poverty now requires a focus
on life events and individual circumstances due to their increased importance. This aligns with the
passage's assertion that "accidents of life" are more significant in the 21st century. Hence, Option (c) is
the correct answer.
(d) Both II and IV
Conclusion II is not supported by the passage. The passage does not claim that socially marginalized
groups are less vulnerable to poverty; it only highlights a shift in the factors influencing poverty.
Conclusion IV is supported by the passage. Hence, Option (d) is not the correct answer.

132. Correct Answer: (b) The training, experience, and combat readiness of pilots significantly influence
the effectiveness of an air force.
Explanation:
(a) Air superiority is primarily determined by the technological advancement and stealth capabilities of
fighter aircraft.
While technology and stealth capabilities are significant factors in determining air superiority, the
passage argues that air power encompasses more than just the number of platforms and their
technological specifications. It includes aspects such as employment tactics, roles, and the experience

Page 46 of 53
and readiness of the aircrew, which this option does not address. Hence, Option (a) is not the correct
answer.
(b) The training, experience, and combat readiness of pilots significantly influence the effectiveness of
an air force.
This explanation aligns with the passage, which emphasizes that true military air power includes factors
beyond just aircraft numbers and technology. The passage highlights the importance of the skill sets,
experience, and combat readiness of the aircrew, making this the most comprehensive explanation.
Hence, Option (b) is the correct answer.
(c) The geographical location and altitude of air bases play a crucial role in determining air power.
While geographical location and altitude can influence air operations, the passage does not focus on
these factors. Instead, it stresses the broader aspects of air power, including tactics, concepts of
operation, and aircrew readiness. Therefore, this option does not fully justify the reasoning in the
passage. Hence, Option (c) is not the correct answer.
(d) The political and economic stability of a nation has a direct impact on its military capabilities.
This option introduces external factors that affect military capabilities but are not directly addressed in
the passage. The passage focuses on internal factors related to air power, such as pilot skills and
operational concepts, making this option less relevant to the argument presented. Hence, Option (d) is
not the correct answer.

133. Correct Answer: (c) Both I and III.


Explanation:
(a) Only I
Conclusion I supports the argument that AI-powered digital assistants could be the "killer app" by
highlighting the seamless integration with other smart devices, which enhances user convenience and
functionality. However, it does not provide comprehensive support compared to the combined evidence
of other options. Hence, Option (a) is not the correct answer.
(b) Only II
Conclusion II suggests increased satisfaction among early adopters of AI-powered smart glasses,
indicating a positive reception. While this is supportive, it alone does not fully establish AI-powered
digital assistants as the "killer app" for smart glasses without additional evidence from other contexts.
Hence, Option (b) is not the correct answer.
(c) Both I and III
Conclusions I and III together provide strong support for the argument. Conclusion I highlights
enhanced functionality and convenience through seamless integration with other devices, while
Conclusion III provides evidence of the effectiveness of AI-powered digital assistants in other
applications, suggesting they could similarly drive the adoption of smart glasses. Hence, Option (c) is
the correct answer.
(d) Both II and IV
Conclusion II supports increased satisfaction among early adopters, but Conclusion IV introduces a
barrier (higher production costs) that limits widespread adoption. While satisfaction is a positive
indicator, the higher costs do not support the notion of AI-powered digital assistants being the "killer
app" if they are not widely adoptable. Hence, Option (d) is not the correct answer.

134. Correct Answer: (a) Music interventions have been consistently shown to reduce symptoms of
anxiety and depression in clinical trials.
Explanation:

Page 47 of 53
(a) Music interventions have been consistently shown to reduce symptoms of anxiety and depression in
clinical trials.
This option strengthens the argument by providing specific evidence that music interventions have
measurable and positive effects on mental health. The reduction of symptoms such as anxiety and
depression supports the claim of improvements in mental health-related quality of life. Hence, Option
(a) is the correct answer.
(b) Artists creating soundscapes often tailor their work to evoke specific emotional responses in
listeners.
While this is related to the artistic aspect of soundscapes, it does not directly provide evidence of the
health benefits of music interventions. It focuses more on the intention behind the creation of
soundscapes rather than their actual impact on mental health. Hence, Option (b) is not the correct
answer.
(c) Music therapy has been incorporated into standard treatment protocols in various mental health
institutions worldwide.
This option suggests that music therapy is recognized and used in practice, but it does not provide direct
evidence of its effectiveness in improving mental health-related quality of life. It indicates acceptance
rather than measurable outcomes. Hence, Option (c) is not the correct answer.
(d) The use of music in therapeutic settings is widely accepted and promoted by leading healthcare
organizations.
Acceptance and promotion by healthcare organizations indicate recognition but do not provide direct
evidence of the effectiveness of music interventions. The argument requires specific evidence of health
benefits rather than general acceptance. Hence, Option (d) is not the correct answer.

135. Correct Answer: (b) All are implicit


Explanation:
(a) Both II and III are implicit
Assumption II is implicit because the statement argues for the prioritization of RCTs based on their
reliability. Assumption III is also implicit as the need for a change in funding priorities suggests that
current practices do not adequately prioritize RCTs. However, this option fails to recognize the
assumption that observational studies can produce misleading findings. Hence, Option (a) is not the
correct answer.
(b) All are implicit
This option is correct because the statement implies that observational studies can produce misleading
findings (I), that RCTs are more reliable (II), and that the current funding practices do not prioritize
RCTs sufficiently (III). All these assumptions are necessary to support the argument for changing
funding priorities. Hence, Option (b) is the correct answer.
(c) Only I is implicit
This option is incorrect because it only acknowledges the potential issues with observational studies,
ignoring the importance of RCTs' reliability and the current funding practices that do not prioritize RCTs
sufficiently. Hence, Option (c) is not the correct answer.
(d) Only II is implicit
This option is incorrect because it fails to recognize the issues with observational studies and the
inadequacies in current funding practices. It only acknowledges the reliability of RCTs without
addressing the other necessary assumptions. Hence, Option (d) is not the correct answer.

136. Correct Answer: (a) Yes, because he stored dangerous chemicals that escaped and caused harm.
Explanation:
Page 48 of 53
(a) Correct. This option directly applies the principle that storing dangerous substances brings with it
an absolute liability for any harm caused by their escape, regardless of any unforeseen or extraordinary
circumstances.
(b) Incorrect. This option introduces a condition not relevant under strict liability, which does not
consider the foreseeability or preventability of the escape event but focuses solely on the fact that the
dangerous substance escaped and caused harm.
(c) Incorrect. Despite the earthquake being an unforeseeable event, the principle of strict liability holds
the owner accountable regardless of the cause of the escape.
(d) Incorrect. This statement is misleading as strict liability in this context does not differentiate
between causes of the escape; the mere fact that the chemicals escaped and caused damage triggers
liability.

137. Correct Answer: (b) No, because the malpractice occurred before her partnership commenced.
Explanation:
(a) Incorrect. Although partners are generally liable for the firm's actions, this liability does not
retroactively extend to actions that took place before their partnership.
(b) Correct. The principle of partnership liability applies to actions during the tenure of the partnership;
since the malpractice occurred before her time, she is not liable.
(c) Incorrect. Liability for actions is limited to the period during which one is a partner. Being a partner
does not automatically extend liability to events prior to joining the firm.
(d) Incorrect. This option misleads by suggesting that her liability could depend on her awareness of
legal issues, which is not relevant under the principle stated. The key factor is the timing of her joining
the firm relative to when the malpractice occurred.
Consider the following principles and respond to the 108. and 109.
Principles:
1. Negligence is the omission to do something which a reasonable man, guided upon those
considerations which ordinarily regulate the conduct of human affairs, would do or doing something
which a prudent or reasonable man would not do.
2. Defendant’s duty of care depends on the reasonable foreseeability of injury which may be caused to
the plaintiff on breach of duty.

138. Correct Answer: (a) Yes, because a reasonable person would have conducted all required safety
checks.
Explanation:
(a) Correct. The principle of negligence involves assessing what a reasonable person would do under
similar circumstances. Given the context of operating a ride, a reasonable person would maintain all
safety protocols to ensure the safety of the riders, making Sarah's failure to perform the safety checks
an act of negligence.
(b) Incorrect. The severity of the injuries does not negate the act of negligence. Negligence focuses on
the breach of a duty of care and the foreseeability of harm, not the severity of the outcome.
(c) Incorrect. Stating that the actions do not amount to negligence due to business distracts from the
core issue. Negligence is determined by the failure to adhere to a standard of care expected in the
circumstances, irrespective of external factors like busyness.
(d) Incorrect. The busyness of the fair does not provide a valid excuse for skipping safety checks.
Negligence is determined based on what a reasonable person would have done, and in this case,
ensuring the safety of ride users is paramount, regardless of external pressures.

Page 49 of 53
139. Correct Answer: (a) No, because the repair was scheduled in a reasonable time frame given the
minimal risk.
Explanation:
(a) Correct. Linda took reasonable measures to address a minor risk by scheduling repairs and marking
the tile. Her actions align with what a reasonable person would do, thus fulfilling her duty of care
without immediate repair.
(b) Incorrect. Simply acknowledging that the tile was uneven does not automatically result in liability,
especially when she assessed the risk as minimal and took reasonable precautions.
(c) Incorrect. The principle of duty of care does not require immediate action in all cases but appropriate
measures based on the risk assessment. Linda’s decision to schedule a repair, given the tile's minimal
risk and its non-central location, was a reasonable measure.
(d) Incorrect. This statement now highlights a specific failure (inadequacy of the signage) rather than
absolving Linda of liability based on the presence of any signage. It emphasizes a potential flaw in how
the risk was communicated, but still concludes that it does not establish negligence because the key
factor is the overall adequacy of her response to the minimal risk.

140. Correct Answer: (c) Yes, because the courier was acting within the scope of employment when the
accident occurred.
Explanation:
(a) Incorrect. This option misrepresents vicarious liability by suggesting it only applies if the employee
was following explicit instructions. Vicarious liability can still apply even if the specific act wasn't
explicitly instructed, as long as it occurred within the scope of employment.
(b) Incorrect. The fact that the maneuver was not explicitly required does not exempt the employer from
liability; the key factor is that the courier was engaged in job-related activities.
(c) Correct. The principle of vicarious liability holds the employer responsible for acts done by the
employee while acting within the scope of employment, even if the specific action was not directly
prescribed.
(d) Incorrect. Whether or not the specific maneuver was part of the official duties, the broader context
shows that the courier was engaged in his employment activities, making the employer liable.

141. Correct Answer: (a) Yes, because the statement about Ms. Clarkson was false and published without
proper verification.
Explanation:
(a) Correct. The principle of defamation clearly states that publishing false and derogatory statements
without lawful justification constitutes defamation. The fact that the newspaper did not verify the
accuracy of the information further solidifies this violation.
(b) Incorrect. The role of reporting on political conduct does not exempt a newspaper from the
obligation to verify the accuracy of the information it publishes. Responsible journalism requires fact-
checking, especially when the statements could damage reputations.
(c) Incorrect. While demonstrating tangible harm might strengthen a defamation case, the essential
violation of defamation law has occurred simply by publishing false information without verification.
Defamation law does not always require proof of harm but focuses on the falsity of the published
statement and the lack of lawful justification.
(d) Incorrect. Belief in the truthfulness of the information does not absolve the newspaper from liability
if the information was not verified. The law requires that reasonable steps be taken to ensure the
accuracy of potentially harmful statements.

Page 50 of 53
142. Correct Answer: (a) Yes, because the noise from the factory interferes with John’s use and
enjoyment of his land.
Explanation:
(a) Correct. The principle states that a private nuisance includes any interference with a person’s use or
enjoyment of land. The loud noise from the factory affects John’s ability to enjoy his farm, thus
constituting a private nuisance.
(b) Incorrect. While some noise is expected, the persistent and excessive nature of the noise, which
affects John’s use and enjoyment of his property, makes it a private nuisance.
(c) Incorrect. Not all noise from a factory is considered a nuisance; it depends on the degree and impact
of the noise on neighboring properties.
(d) Incorrect. While Kevin has the right to operate his factory, this right is not absolute and does not
permit activities that unreasonably interfere with his neighbors’ enjoyment of their land.

143. Correct Answer: (d) No, because the court cannot enforce any rights arising from an illegal act.
Explanation:
(a) Incorrect. Despite the violation of their agreement, the entire operation is illegal, and thus, any
agreements made in furtherance of that illegal activity cannot be legally enforced.
(b) Incorrect. This option redirects from the primary legal principle that rights cannot arise from illegal
activities by focusing on a narrower aspect of the profit-sharing dispute, but it does not address the
underlying illegality that precludes any legal claims.
(c) Incorrect. Even if Mr. Thompson could prove the sale was agreed upon, the underlying activity
remains illegal, and thus no legal claim can be founded on it.
(d) Correct. The principle clearly states that no legal action can arise from an illegal activity. Since the
agreement between Mr. Thompson and Mr. Green involves illegal smuggling, any breach of this
agreement is unenforceable in court.

144. Correct Answer: (c) Youngers facing employment challenges would be primarily due to their
perceived lack of experience.
Explanation:
(a) Correct. Companies might initiate such programs as a countermeasure to the stereotype of Elders
being less technologically savvy.
(b) Correct. This could be a strategy employed by Elders to bypass age-related stereotypes affecting
employment opportunities.
(c) Incorrect. The passage indicates that Youngers are favored for their adaptability and fluency with
technology, suggesting their employment challenges would not be due to a lack of experience but
perhaps other factors not covered by the stereotype.
(d) Correct. Given the stereotypes of Elders, companies might prefer them for roles that value their
experience over adaptability, which aligns with societal perceptions.

145. Correct Answer: (c) New policies are introduced focusing equally on youth-related issues like job
security and climate change.
Explanation:
(a) Correct. This could be seen as a strategic move by Elders to maintain their influence by appearing to
address the needs of Youngers while securing their own interests.
(b) Correct. Lowering age requirements could be a response to pressures for more inclusive
representation but does not guarantee significant shifts in power dynamics.

Page 51 of 53
(c) Incorrect. Given the Elders' political dominance and their focus on issues pertinent to their age
group, it's less likely that there would be an equal focus on youth-related issues unless there is a
substantial shift in political power.
(d) Correct. This outcome is likely considering the current dominance of Elders in politics and their
vested interest in maintaining focus on issues that directly benefit them.

146. Correct Answer: (a) The focus of government policies remains heavily skewed towards the
interests of Elders, such as pension security and healthcare.
Explanation:
(a) Correct. The passage suggests that with Elders' continued dominance, policies will likely continue
favoring their interests, maintaining the status quo.
(b) Unlikely. Given the established dominance of Elders and their perceived stability and experience, a
dramatic shift in power to younger politicians would require significant changes beyond current trends.
(c) Possible. This could happen as Youngers push for societal changes, though it does not guarantee a
shift in actual power or immediate policy changes.
(d) Unlikely. The rapid redistribution of political power is improbable given the existing structures and
biases favoring Elders.

147. Correct Answer: (b) The emergence of new political movements led by Youngers, aimed at
restructuring the age-based power dynamics.
Explanation:
(a) Correct. Political apathy could increase among Youngers if they feel their efforts are futile, potentially
leading to decreased engagement in voting.
(b) Correct. The passage indicates a high level of frustration among Youngers, which could fuel the
formation of new political movements seeking to change the current age-based inequalities.
(c) Unlikely. Given the described frustrations and ongoing advocacy efforts, it's improbable that
Youngers would suddenly disengage completely.
(d) Unlikely. The strong sentiment and active movements mentioned suggest sustained interest and
engagement, not dissolution.

148. Correct Answer: (c) Yes, because there was a total restraint without just cause or legal excuse.
Explanation:
(a) Incorrect. While Mr. Bennett did not have direct evidence, the key issue is the justification of the total
restraint, not just the absence of direct evidence.
(b) Incorrect. Reasonable suspicion alone does not justify total restraint without additional evidence or
legal procedure, which Mr. Bennett lacked.
(c) Correct. The principle specifies that false imprisonment involves confinement without just cause or
excuse, and Mr. Bennett confined Ms. Turner based solely on suspicion and without any direct evidence,
fulfilling the criteria for false imprisonment.
(d) Incorrect. Protecting one's property does not justify confining an individual without clear evidence
of a crime being committed, especially in a manner that totally restrains their freedom without legal
authority.

149. Correct Answer: (a) Yes, because they acted together in vandalizing the property.
Explanation:

Page 52 of 53
(a) Correct. The principle clearly states that individuals can be joint tort-feasors if they engage in a
common action that results in wrongdoing. Alex and Jamie acting together to vandalize the building
establishes them as joint tort-feasors.
(b) Incorrect. The severity of the damage does not negate the fact that a tort was committed. Joint
liability does not depend on the extent of damage but on the joint action leading to the tort.
(c) Incorrect. Although the vandalism occurred during a protest, the context does not excuse them from
liability for their actions.
(d) Incorrect. Believing that one is exercising free speech rights does not provide a legal justification for
committing a tort such as vandalism.

150. Correct Answer: (b) No, because she withdrew from the action before it was carried out.
Explanation:
(a) Incorrect. Although Linda was involved in the initial planning, joint and several liabilities require
participation in the actual wrongful act. Planning alone does not establish liability without participation
in the act.
(b) Correct. The principle stipulates joint and several liabilities for those who partake in the tortious
act. Linda's withdrawal before the act was carried out exempts her from liability, as she did not
participate in or contribute to the actual tort committed.
(c) Incorrect. Being part of the planning does not automatically result in liability unless one participates
in the wrongful act itself. Mere inclusion in planning discussions is insufficient for liability if there is no
participation in the actual tortious activity.
(d) Incorrect. This statement attempts to shift focus to the direct causation of damage, which is
irrelevant under the principle of joint tort-feasors. Liability in this context depends on participation in
the act, not direct causation. This answer could confuse the issue by suggesting that liability depends
on a direct cause rather than participation in the wrongful act itself.

Page 53 of 53

You might also like